Sunteți pe pagina 1din 309

BAZELE

ELECTROTEHNICII I

Conf. Dr. Ing. Veronica PĂLTÂNEA

Bazele Electrotehnicii I - Conf. dr. Ing. Veronica pALTÂNEA, Facultatea de Inginerie Electrică 1
Punctaj - 100 puncte
Examen final - 50 puncte
Teorie - 20 puncte
Probleme - 30 puncte
Problema 1-10 puncte
Problema 2-20 puncte

Parcurs - 50 puncte
Lucrare - 30 puncte (săptămâna 8, 9)
Activitate seminar - 10 puncte (prezență, teme de casă)
Activitate curs - 10 puncte (caiet de curs, prezență)

Bazele Electrotehnicii I - Conf. dr. Ing. Veronica PÂLTÂNEA, Facultatea de Inginerie Electrică
Bazele Electrotehnicii I
Curs
Introducere. Mărimi electromagnetice
Legile teoriei macroscopice a electromagnetismului
Regimul electrostatic al câmpului electromagnetic
Regimul electrocinetic al câmpului electromagnetic
Regimul electrodinamic al câmpului electromagnetic
Seminar
Circuite electrice de curent continuu
Regimul electrostatic
Regimul electrodinamic

Bazele Electrotehnicii I - Conf. dr. Ing. Veronica PĂLÎÂNEA, Facultatea de Inginerie Electrică
Curs 1 Introducere. Mărimi electromagnetice

1.1. Introducere. Regimuri de desfășurare a fenomenelor


electromagnetice
1.2. Mărimi electromagnetice primitive și derivate
1.2.1. Clasificare
1.2.2. Starea de încărcare electrică și câmpul electric
1.2.3. Starea de polarizare electrică
1.2.4. Starea de magnetizare și câmpul magnetic
1.2.5. Starea electrocinetică
1.2.6. Câmpul electric imprimat

Bazele Electrotehnicii I - Conf. dr. Ing. Veronica PĂLTÂNEA, Facultatea de Inginerie Electrică
1.1. Introducere. Regimuri de desfășurare a fenomenelor electromagnetice

1.1.1. Conceptul de câmp electromagnetic


Experiența a arătat că există stări ale corpurilor pentru caracterizarea cărora descrierea lor
parametrică (geometrică) mecanică, termică și chimică este insuficientă.
Tot experiența a arătat că, atunci când se află în asemenea stări, între corpuri se exercită
de la distanță unele interacțiuni (forțe, cupluri) numite electromagnetice, ireductibile la
cele de natură mecanică.
Conform concepției moderne, aceste interacțiuni la distanță sunt efectul acțiunii exercitate
asupra corpurilor de un sistem fizic distinct - câmpul electromagnetic.
Acesta este o formă de existență a materiei care poate apărea atât în afara cât și în
interiorul corpurilor.
Câmpul electromagnetic se propagă din aproape în aproape (prin contiguitate) cu o viteză
foarte mare, dar finită (viteza luminii în mediul respectiv).

Bazele Electrotehnicii I - Conf. dr. Ing. Veronica PÂLTÂNEA, Facultatea de Inginerie Electrică S
Câmpul electric și câmpul magnetic reprezintă aspecte complementare indisolubil legate
ale câmpului electromagnetic și, în regim variabil, se condiționează reciproc.
Corpul aflat în stări electrice și magnetice specifice își asociază un câmp electric sau
magnetic propriu, iar acesta, va exercita asupra oricărui alt corp aflat într-o stare similară
acțiuni ponderomotoare corespunzătoare.
Câmpul electromagnetic este deci purtător de energie și impuls.

Electrotehnica reprezintă studiul fenomenelor electrice și magnetice din punctul de vedere


al aplicațiilor lor tehnice.
Desemnează în același timp și acea ramură a industriei și tehnicii care are drept obiect
proiectarea, construcția și exploatarea utilajelor și aparaturii realizate pe baza utilizării
fenomenului electromagnetic.
Teoria fenomenologică a electromagnetismului reprezintă baza teoretică pentru studiul
electrotehnicii.

Bazele Electrotehnicii I - Conf. dr. Ing. Veronica PÂLTÂNEA, Facultatea de Inginerie Electrică 6
1.1.2. Regimurile câmpului electromagnetic
în teoria macroscopică a câmpului electromagnetic se cunosc patru regimuri, clasificate după
modul de variație al mărimilor electromagnetice:

Regimul static: mărimile fizice nu variază în timp și nu există fenomene termodinamice (nu
există schimburi și transformări de energie). Acesta este singurul regim în care fenomenele
electrice și magnetice se pot produce și studia independent sub forma regimului
electrostatic, respectiv a regimului magnetostatic.

Regimul staționar: mărimile rămân invariabile în timp, dar apar schimburi și transformări
energetice (exemplu: regimul de curent continuu) astfel încât se identifică regimul electric
staționar, respectiv regimul magnetic staționar.

Regimul cvasistaționar: mărimile variază în timp, dar cu o viteză (frecvență) care permite
neglijarea producerii, transmiterii și recepției undelor electromagnetice.

Regimul nestaționar (general): este regimul în care nu se fac neglijări ale mărimilor
câmpului electromagnetic. Descrie cea mai generală formă de variație în timp a mărimilor,
în care intervine fenomenul de radiație electromagnetică.

Bazele Electrotehnicii I - Conf. dr. Ing. Veronica PÂLTÂNEA, Facultatea de Inginerie Electrică 7
1.2. Mărimi electromagnetice primitive și derivate

1.2.1. Clasificare
După modul de introducerea mărimilor
mărimi primitive, se introduc pe cale inductivă făcând direct apel la experiență
mărimi derivate, se exprimă matematic cu ajutorul mărimilor primitive pentru o
mai bună caracterizare a unui fenomen fizic.
în orice teorie, doar numărul mărimilor primitive este constant. în teoria câmpului
electromagnetic mărimile primitive sunt:
q sarcina electrica descrie starea de încărcare a coipurilor;

p momentul electric descrie starea de polarizare electrică a coipurilor;

/ intensitatea curentului electric de conducție caracterizează starea electrocinetică (de conducție) a coipurilor:

in momentul magnetic descrie starea de magnetizare a coipurilor;

£v vectorul câmp electric în vid este componenta electrică a câmpului electromagnetic:

By vectorul inducție magnetica în vid este componenta magnetică a câmpului electromagnetic.

Bazele Electrotehnicii I - Conf. dr. Ing. Veronica PÂLTÂNEA, Facultatea de Inginerie Electrică 8
în orice teorie de câmp este necesar a se realiza o dublă descriere a fenomenelor în raport
cu anumite curbe, suprafețe sau volume. Astfel mărimile electromagnetice se clasifică în:
mărimi locale (punctuale),
mărimi globale (integrale).

NOTAȚII UTILIZATEÎN CURS


Pentru descrierea unei curbe deschise, definită între punctele A, respectiv B, se notează
curba cu (C) și este definită prin elementul de lungime dl, care este un vector orientat;

Curbă deschisă Curbă închisă

Pentru descrierea unei curbe închise (A=B) se notează curba cu (T).

Lungimea unei curbe esteL =


Bazele Electrotehnicii I - Conf. dr. Ing. Veronica PÂLTÂNEA, Facultatea de Inginerie Electrică 9
Pentru descrierea unei suprafețe deschise, mărginită de curba închisă (T), se notează
suprafața cu (Sr) și este definită prin elementul de suprafață cL4.
Se definește un vector orientat după normala la suprafață cU = ncU .

Suprafață
deschisă

Pentru a definii o suprafață închisă se notează cu


Obs.: suprafața închisă nu trebuie să fie neapărat o sferă.
:ă închisă
Aria unei suprafețe se calculează:

Bazele Electrotehnicii I - Conf. dr. Ing. Veronica PÂLTÂNEA, Facultatea de Inginerie Electrică 10
Pentru descrierea unui volum, mărginit de suprafața închisă (Z), se notează cu (Dv) și este
definit prin elementul de volum dV.

Obs.: suprafața închisă nu trebuie să fie neapărat o sferă.

Bazele Electrotehnicii I - Conf. dr. Ing. Veronica PÂLTÂNEA, Facultatea de Inginerie Electrică 11
Din punctul de vedere al sistemului de unități de măsură:

fundamental

secundar

sistemul MKSA raționalizat (SI): metru, kilogram, secundă, ampere

sistemul CGS: centimetru, gram, secundă

Conform principiului cauzalității sunt:

mărimi de stare ce caracterizează starea unui sistem fizic la un moment dat

mărimi de proces

Din punct de vedere matematic sunt:

mărimi scalare

mărimi vectoriale

mărimi tensoriale
Bazele Electrotehnicii I - Conf. dr. Ing. Veronica PÂLTÂNEA, Facultatea de Inginerie Electrică 12
1.2. Mărimi electromagnetice primitive și derivate

1.2.2. Starea de încărcare electrică și câmpul electric


A. Sarcina electrică și câmpul electricîn vid
Pentru a investiga simultan starea de încărcare și câmpul electric se imaginează o experiență
care folosește un corp de probă metalic sau metalizat de mici dimensiuni, plasat în vid și
încărcat cu sarcina electrică q. Se constată că:
în același punct al câmpului asupra unui corp de probă cu stări de încărcare diferite se
exercită forțe având o direcție unică, dar diferind ca mărime și sens;
raportul în care se găsesc forțele care se exercită asupra a două corpuri de probă cu stări
de încărcare diferite este același în orice punct al câmpului.

Bazele Electrotehnicii I - Conf. dr. Ing. Veronica PÂLTÂNEA, Facultatea de Inginerie Electrică 13
Forța care se exercită asupra corpului de probă este denumită forța Coulomb și are
expresia:
Fe= qEv
Se introduc simultan două mărimi primitive:

Sarcina electrică q, este o mărime fizică scalară care caracterizează global starea de
încărcare a corpului de probă. Este o mărime fizică extensivă: la alipirea / divizarea unor
corpuri încărcate sarcina lor electrică se adună / divide scalar; unitatea de măsură este
coulomb [C].

Vectorul Ev intensitate câmp electric în vid este o mărimea fizică vectorială ce


caracterizează local câmpul electric în vid; unitatea de măsură este volt/metru [V/m].

Bazele Electrotehnicii I - Conf. dr. Ing. Veronica PÂLTÂNEA, Facultatea de Inginerie Electrică 14
B. Distribuții de sarcini electrice
în cazul corpurilor masive, caracterizarea stării de încărcare electrică se face punctual
considerând un set de mărimi locale, numite densități de sarcină electrică.
Densitatea de volum a sarcinii electrice
Se notează cu și se definește ca raportul:

Unitatea de măsură este coulomb/metru-cub [C/m3].


Sarcina electrică totală dintr-un domeniu (Dv) este dată de integrala de volum:

Bazele Electrotehnicii I - Conf. dr. Ing. Veronica PÂLTÂNEA, Facultatea de Inginerie Electrică 15
Densitatea de suprafață a sarcinii electrice
Se notează cu ps și se definește ca raportul:
.. Aq dq
pț = lim — = —L
s AA-»0 AA dA

Unitatea de măsură este coulomb/metru-pătrat [C/m2].


Sarcina electrică totală de pe o suprafață (Sr) este dată de integrala de suprafață:

Bazele Electrotehnicii I - Conf. dr. Ing. Veronica PÂLTÂNEA, Facultatea de Inginerie Electrică 16
Densitatea de linie a sarcinii electrice
Se notează cu p; și se definește ca raportul:

Unitatea de măsură este coulomb/metru [C/m].


Sarcina electrică totală distribuită pe o curbă (C) este dată de integrala de linie:

Bazele Electrotehnicii I - Conf. dr. Ing. Veronica PÂLTÂNEA, Facultatea de Inginerie Electrică 17
C. Câmpul electricîn corpuri masive
Câmpul electric reprezintă un aspect particular al sistemului fizic numit câmp electromagnetic.
Un corp masiv plasat în vid nu mai poate fi caracterizat local doar printr-o singură mărime electrică
Ev ca în cazul corpului punctual, astfel încât este necesară definirea mărimilor electrice locale,
vectoriale:

Intensitatea câmpului electric


Se notează cu E care coincide în vid cu Ev
Unitatea de măsură este volt/metru [V/m].
Inducția electrică
Se notează cu D, care în vid se definește D = e0E
Unitatea de măsură este coulomb/metru-pătrat [C/m2]

Se definește constanta universală permitivitatea electrică a vidului

Unitatea de măsură este farad/metru [F/m]

Bazele Electrotehnicii I - Conf. dr. Ing. Veronica PÂLTÂNEA, Facultatea de Inginerie Electrică 18
D. Mărimi de caracterizare globală a câmpului electric
Asociat celor două mărimi electrice locale și vectoriale (E, respectivi)), se definesc două mărimile
globale, scalare.

Tensiunea electrică

Se notează cu u
Unitatea de măsură este volt [V].

Prin definiție liniile de câmp ale vectorului E se numesc //n/7 de câmp electric. Acest vector
este tangent la liniile de câmp în fiecare punct al lor.

Tensiunea electromotoare se definește ca:

Bazele Electrotehnicii I - Conf. dr. Ing. Veronica PÂLTÂNEA, Facultatea de Inginerie Electrică 19
Fluxul electric
Se notează cu Ψ

Unitatea de măsură este coulomb [C]


Observații:
Sensul de integrare arbitrar ales se numește sens de referință. Sensul de referință care
corespunde valorii pozitive a mărimii se numește sens fizic;
în cazul suprafețelor închise, normala se consideră întotdeauna orientată către exteriorul
suprafeței;

în cazul suprafețelor deschise, după stabilirea unui sens de parcurgere a curbelor închise
care le mărginește, se folosește regula burghiului drept.
Bazele Electrotehnicii I - Conf. dr. Ing. Veronica PÂLTÂNEA, Facultatea de Inginerie Electrică 20
1.2. Mărimi electromagnetice primitive și derivate
1.2.3. Starea de polarizare electrică
Se identifică a doua stare de electrizare a corpurilor, în afara celei de încărcare. Experiența arată că
există corpuri neîncărcate electric asupra cărora un câmp electric extern exercită acțiuni
ponderomotoare și care, la rândul lor, își asociază un câmp electric propriu. Această nouă stare se
numește stare de polarizare electrică ș\ poate fi obținută în două moduri:
la introducerea unui corp neconductor într-un câmp electric exterior. Dacă la înlăturarea câmpului
electric corpul revine la starea nepolarizată, atunci apare fenomenul de polarizare temporară, iar
corpurile având această proprietate se numesc dielectrici.

ca urmare a unor cauze de natură neelectrică independente de existenta unui câmp electric
exterior. Acest tip de polarizare se numește polarizare permanentă și poate apărea în urma
încălzirii sau deformării mecanice a unor cristale sau a solidificării în câmpuri electrice puternice a
unor substanțe topite, polarizabile (electreți). Cele două tipuri de polarizare pot coexista,
neexcluzându-se reciproc.
Bazele Electrotehnicii I - Conf. dr. Ing. Veronica PÂLTÂNEA, Facultatea de Inginerie Electrică 21
A. Caracterizare globală
Un mic corp punctual polarizat, caracterizat prin momentul electric p, plasat în vid, este
introdus într-un câmp electric exterior, astfel încât asupra lui acționează un cuplu de rotație
dat de expresia:
C = p x Ev
și/sau o forță de deplasare având expresia:

F = (pgrad)Ev

Unitatea de măsură a momentului electric este coulomb-metru [Cm]


Momentul electric este suma momentelor temporar și permanent
P = Pt + Pp

Bazele Electrotehnicii I - Conf. dr. Ing. Veronica PÂLTÂNEA, Facultatea de Inginerie Electrică 22
B. Caracterizare locală
Un corp masiv caracterizat global prin momentul electric p, plasat în vid, introdus într-un
câmp electric exterior, poate fi caracterizat local prin densitatea de volum a momentului
electric. Se definește mărimea locală și vectorială, polarizație electrică, notată P, care are
expresia:

P=

Unitatea de măsură a polarizației electrice este coulomb/metru-pătrat [C/m2]


Polarizația electrică este suma polarizației temporare și a polarizației permanente

P = Pt + PP

Starea de polarizație se poate obține prin:


tratamente speciale precum: deformare mecanică (piezoelectricitate), încălzire
(piroelectricitate), topire și solidificare într-un câmp electric intens (electreți);
simpla introducere a lor într-un câmp electric.

Bazele Electrotehnicii I - Conf. dr. Ing. Veronica PÂLTÂNEA, Facultatea de Inginerie Electrică 23
https://wlki.metropolia.fi/display/sensor/Piezoelectric+sensing

https://em.geoscl.xyz/content/physical_properties/dielectric_permittivrty/index.html

http://vavw.electricaltech4u.com/electric-polarization/ https://www.priyamstudycentre.com/2020/02/electric-polarization.html

Bazele Electrotehnicii I - Conf. dr. Ing. Veronica PÂLTÂNEA, Facultatea de Inginerie Electrică 24
1.2. Mărimi electromagnetice primitive și derivate
1.2.4. Starea de magnetizare și câmpul magnetic
A. Sarcina electrică și câmpul magnetic în vid
Din experiență s-a observat că există corpuri ne-conductoare electric care plasate în câmp magnetic
dezvoltă interacțiuni speciale, altele decât cele de natură electrică. Această stare se definește ca fiind starea
de magnetizare.
Interacțiunile dintre corpuri se numesc magnetice, iar sistemul fizic asociat corpurilor magnetice și prin
intermediul cărora se transmit aceste interacțiuni se numește câmp magnetic.
Câmpul magnetic este generat de:
corpurile magnetizate,

conductoareleîn stare electrocinetică și corpurile electrizateîn mișcare,

variația rapidăîn timp a câmpului electric.

Câmpul magnetic exercită acțiuni asupra corpurilor magnetizate și corpurilor în stare electrocinetică.

Bazele Electrotehnicii I - Conf. dr. Ing. Veronica PÂLTÂNEA, Facultatea de Inginerie Electrică 25
Se consideră un corp punctual plasat în vid și încărcat cu sarcina electrică q. Acesta se
deplasează cu viteza vîn câmp magnetic.
Forța care se exercită asupra corpului de probă este denumită forța Lorentz și are
expresia:

Se introduce mărimea primitivă:

Vectorul Bv inducția magnetică în vid este o mărimea fizică vectorială care


caracterizează local câmpul magnetic în vid; unitatea de măsură este tesla [T].

Bazele Electrotehnicii I - Conf. dr. Ing. Veronica PÂLTÂNEA, Facultatea de Inginerie Electrică 26
B. Câmpul magnetic în corpuri masive
Câmpul magnetic reprezintă un aspect particular al sistemului fizic numit câmp electromagnetic.
Un corp masiv plasat în vid nu mai poate fi caracterizat local doar printr-o singură mărime magnetică
Bv ca în cazul corpului punctual, astfel încât este necesară definirea mărimilor electrice locale,
vectoriale:

Inducția magnetică
Se notează cu B care coincide în vid cu Bv
Unitatea de măsură este tesla [T].
Intensitatea câmpului magnetic
Se notează cu H, care în vid se definește B = p0H
Unitatea de măsură este amper/metru [A/m]
Se definește constanta universală permeabilitatea magnetică a vidului p0 = 4π*10-7 [H/m]
Unitatea de măsură este henry/metru [H/m]

Bazele Electrotehnicii I - Conf. dr. Ing. Veronica PÂLTÂNEA, Facultatea de Inginerie Electrică 27
C. Mărimi de caracterizare globală a câmpului magnetic
Asociat celor două mărimi magnetice locale și vectoriale (H, respectiv B), se definesc două
mărimile globale, scalare.

Tensiunea magnetică
Se notează cu uvlAB
Unitatea de măsură este amper [A]

Prin definiție liniile de câmp ale vectorului H se numesc linii de câmp magnetic. Acest
vector este tangent la liniile de câmp în fiecare punct al lor.

Tensiunea electromotoare se definește ca:

Bazele Electrotehnicii I - Conf. dr. Ing. Veronica PÂLTÂNEA, Facultatea de Inginerie Electrică 28
Fluxul magnetic
Se notează cu

Unitatea de măsură este weber [Wb]


Observații:
Sensul de integrare arbitrar ales se numește sens de referință. Sensul de referință care
corespunde valorii pozitive a mărimii se numește sens fizic;
în cazul suprafețelor închise, normala se consideră întotdeauna orientată către exteriorul
suprafeței;

în cazul suprafețelor deschise, după stabilirea unui sens de parcurgere a curbelor închise
care le mărginește, se folosește regula burghiului drept.
Bazele Electrotehnicii I - Conf. dr. Ing. Veronica PÂLTÂNEA, Facultatea de Inginerie Electrică 29
D. Starea de magnetizare
Experiența arată că există corpuri magnetizate, care își asociază un câmp magnetic propriu și corpuri
nemagnetizate asupra cărora un câmp magnetic extern exercită acțiuni ponderomotoare. Această
nouă stare se numește stare de magnetizare și poate fi obținută în două moduri:

la introducerea unui corp magnetizabil într-un câmp magnetic exterior. Dacă la înlăturarea
câmpului magnetic corpul revine la starea inițială, atunci apare fenomenul de magnetizare
temporară

ca urmare a unei structuri microscopice a corpului de probă independente de existenta unui câmp
magnetic exterior. Acest stare se definește ca magnetizare permanentă și poate apărea în magneți
permanenți sau datorită deformării mecanice a unor cristale sau a solidificării în câmpuri
magnetice puternice a unor substanțe topite

Bazele Electrotehnicii I - Conf. dr. Ing. Veronica PÂLTÂNEA, Facultatea de Inginerie Electrică 30
Caracterizare globală a stării de magnetizare
Un mic corp punctual magnetizat, caracterizat prin momentul magnetic m, plasat în vid, este
introdus într-un câmp magnetic exterior, astfel încât asupra lui acționează un cuplu de rotație
dat de expresia:
C = m x Bv
și/sau o forță de deplasare având expresia:

F = (mgrad)Bv

Unitatea de măsură a momentului magnetic este amper-metru-pătrat [Am2]


Momentul magnetic este suma momentelor temporar și permanent
m = mt + mp

Bazele Electrotehnicii I - Conf. dr. Ing. Veronica PÂLTÂNEA, Facultatea de Inginerie Electrică 31
Caracterizarea locală a stării de magnetizare
Un corp masiv caracterizat global prin momentul magnetic m, plasat în vid, introdus într-un
câmp magnetic exterior, poate fi caracterizat local prin densitatea de volum a momentului
magnetic. Se definește mărimea locală și vectorială, magnetizație, notată M, care are
expresia:

hm _ dm
M=

Unitatea de măsură a magnetizației este amper/metru [A/m]


Magnetizația este suma magnetizației temporare și a magnetizației permanente
/--------------------------------------------------

M = M t + Mp

Bazele Electrotehnicii I - Conf. dr. Ing. Veronica PÂLTÂNEA, Facultatea de Inginerie Electrică 32
http://caseygarrettphysic$.blogspot.conV2013/04/standard-63-permanent-and-temporary.html
https://ro.m.wlkipedia.org/wikl/Fi%C8%99ler:Electric_motor.gif

Bazele Electrotehnicii I - Conf. dr. Ing. Veronica PÂLTÂNEA, Facultatea de Inginerie Electrică 33
1.2. Mărimi electromagnetice primitive și derivate

1.2.5. Starea electrocinetică


Experiența arată că există o stare specifică numai corpurilor conductoare în care acestea suferă acțiuni din
partea câmpului electromagnetic, numită stare electrocinetică. Aceasta este determinată de prezența unui
câmp electric în conductoare și se identifică prin efectele care o însoțesc. Aceste efecte sunt:
magnetice: conductoarele aflate în această stare își asociază un câmp magnetic;
optice: apar descărcări în gaze rarefiate, sau indirect ca urmare a încălzirii până la incandescență;
electrodinamice: se exercită forțe din partea câmpului electromagnetic asupra conductoarelor aflate în stare
electrocinetică, forțe diferite de cele electrice și magnetice;
termice: starea electrocinetică este însoțită de degajare de căldură;
magnetoelectrice: când sarcina electrică a unor conductoareîncărcate variază;
chimice: la unele conductoare starea electrocinetică este însoțită de reacții chimice; sub acest aspect deosebim:
conductoare de speța I în care nu există efecte chimice, iar conducția este electronică

conductoare de speța II (electroliții) care asigură o conducție ionică.

Bazele Electrotehnicii I - Conf. dr. Ing. Veronica PÂLTÂNEA, Facultatea de Inginerie Electrică 34
A. Caracterizare globală
Se consideră un conductor electric având o lungime / parcurs de curentul electric de conducție i, plasat
în vid într-un câmp magnetic de inducție Br. Asupra conductorului se va exercita o forță de către
câmpul magnetic, numită forța Laplace, care are expresia:

AF = i&l x Bv

unde mărimile sunt:


Intensitatea curentului electric de conducție zeu unitatea de măsură amper [A]
Inducția magnetică în vid B} cu unitatea de măsură tesla [T]

Bazele Electrotehnicii I - Conf. dr. Ing. Veronica PÂLTÂNEA, Facultatea de Inginerie Electrică 35
B. Caracterizare locală
în cazul unei repartiții de volum a sarcinii electrice, pentru caracterizarea locală a stării electrocinetice,
se folosește o mărime vectorială derivată J numită densitate de curent, definită astfel încât fluxul său
printr-o suprafață oarecare să fie egal cu intensitatea curentului de conducție prin acea suprafață.

Densitatea de curent J
Mărime fizică derivată, vectorială, de caracterizare locală
Unitatea de măsură amper/metru-pătrat [A/m2]
Liniile de câmp ale vectorului Jse numesc linii de curent
Conductoarele sunt considerate filiforme, când dimensiunea lor transversală este atât de redusă încât,
densitatea curentului prin secțiune poate fi considerată uniformă.
în acest caz curentul electric de conducție se scrie

Bazele Electrotehnicii I - Conf. dr. Ing. Veronica PÂLTÂNEA, Facultatea de Inginerie Electrică 36
https://quickfield.com/glossary/electric_motor_design.htm https://www.integratedsoft.com/Products/magneto.aspx

Bazele Electrotehnicii I - Conf. dr. Ing. Veronica PÂLTÂNEA, Facultatea de Inginerie Electrică 37
1.2. Mărimi electromagnetice primitive și derivate
1.2.6. Câmpul electric imprimat
în conductoare există uneori fenomene de natură neelectrică care produc efecte similare celor pe care le-ar
produce un câmp electric. Câmpul electric fictiv care produce efecte identice cu cele de natură neelectrică
se numește câmp electric imprimat.
Acesta se poate localiza în volumul sau pe suprafața de contact a corpurilor conductoare și se caracterizează
local prin mărimea derivată vectorială numită intensitatea câmpului electric imprimat, notată E,. Unitatea
de măsură volt/metru [V/m].
Caracterizarea globală a câmpului electric imprimat se face în raport cu o curbă închisă prin definirea
mărimii tensiune electromotoare imprimată:

Unitatea de măsură este volt [V]

Bazele Electrotehnicii I - Conf. dr. Ing. Veronica PÂLTÂNEA, Facultatea de Inginerie Electrică 38
Exemple
Câmpul imprimat de accelerație
Dacă se rotește un disc metalic în jurul axului său electronii sunt atrași de periferia discului, iar centrul acestuia rămâne încărcat pozitiv.
Apare astfel un câmp electric imprimat dirijat dinspre periferie spre centrul discului.

Câmpul imprimat de concentrație


Dacă într-un vas un perete poros desparte o zonă 1 cu o soluție de acid clorhidric concentrat, de altă zonă 2 cu acid clorhidric diluat,
ionii componenți ai acidului din prima zonă tind să treacă prin perete în cea de a doua zonă. Experiența arată că ionii pozitivi de
hidrogen H~ având o mobilitate mai mare decât cei negativi de clor Cl- încarcă pozitiv acidul din cea de a doua zonă și negativ pe cel din
a doua. Se formează astfel un câmp electric imprimat dirijat dinspre zona 1 spre zona 2.

Câmpul imprimat termoelectric


Dacă se încălzește o bară conductoare în mod diferit la cele două capete ale sale electronii se deplasează din zona cu agitație termică
mai mare de temperatură spre extremitatea opusă de temperatură 7,. Câmpul electric imprimat este dirijat în sensul opus deplasării
electronilor.

Bazele Electrotehnicii I - Conf. dr. Ing. Veronica PÂLTÂNEA, Facultatea de Inginerie Electrică 39
Mărimi de caracterizare a stării electromagnetice a corpurilor

Starea de electrizare
Sarcina electrică q [C]
Densitatea de volum a sarcinii electrice pv [C/m3];
Densitatea de suprafață a sarcinii electrice p5 [C/m2];
Densitatea de linie a sarcinii electrice pl [C/m];

Starea de polarizare electrică


Momentul electricp [Cm]
Polarizația P [C/m2]

Bazele Electrotehnicii I - Conf. dr. Ing. Veronica PÂLTÂNEA, Facultatea de Inginerie Electrică 40
Mărimi de caracterizare a stării electromagnetice a corpurilor

Starea de magnetizare
Momentul magnetic w [Am2]
Magnetizația M [A/m]

Starea electrocinetică
Curentul electric de conducție i [A]
Densitatea curentului electric J [A/m2]

Bazele Electrotehnicii I - Conf. dr. Ing. Veronica PÂLTÂNEA, Facultatea de Inginerie Electrică
Mărimi de caracterizare a câmpului electromagnetic

Câmpul electric
Mărimi locale vectoriale
Intensitatea câmpului electric E [V/m]
Inducția electrică D [C/m2]

Mărimi globale scalare


Tensiunea electrică u [V]
Fluxul electric Ψ [C]

Bazele Electrotehnicii I - Conf. dr. Ing. Veronica PÂLTÂNEA, Facultatea de Inginerie Electrică 42
Mărimi de caracterizare a câmpului electromagnetic

Câmpul magnetic
Mărimi locale vectoriale
Intensitatea câmpului magneticH [A/m]
Inducția magneticăB [T]

Mărimi globale scalare


Tensiunea magnetică um [A]
Fluxul magnetic Φ [Wb]

Bazele Electrotehnicii I - Conf. dr. Ing. Veronica PÂLTÂNEA, Facultatea de Inginerie Electrică 43
BAZELE
ELECTROTEHNICII I

Conf. Dr. Ing. Veronica PÂLTÂNEA

Bazele Electrotehnicii I - Conf. dr. ing. Veronica PÂLTÂNEA, Facultatea de Inginerie Electrică 1
Bazele Electrotehnicii I
Curs
Introducere. Mărimi electromagnetice
Legile teoriei macroscopice a electromagnetismului
Regimul electrostatic al câmpului electromagnetic
Regimul electrocinetic al câmpului electromagnetic
Regimul electrodinamic al câmpului electromagnetic
Seminar
Circuite electrice de curent continuu
Regimul electrostatic
Regimul electrodinamic

Bazele Electrotehnicii I - Conf. dr. Ing. Veronica PÂLTÂNEA, Facultatea de Inginerie Electrică
Curs 2 Legile teoriei macroscopice a
electromagnetismului (Teoria Maxwell - Hertz)

2.1. Introducere 2.4. Legi generale


2.2. Legi de material 2.4.1. Legea fluxului electric
2.2.1. Legea polarizației temporare 2.4.2. Legea fluxului magnetic
2.4.3. Legea inducție electromagnetice
2.2.2. Legea magnetizației temporare
2.4.4. Legea circuitului magnetic
2.2.3. Legea conducției electrice
2.4.5. Legea conservării sarcinii electrice
2.3. Legi de legătură 2.4.6. Legea transformărilor energetice
2.3.1. Legea legăturii în câmp electric în conductoarele în stare electrocinetică
2.3.2. Legea legăturii în câmp magnetic 2.5. Legea electrolizei

Bazele Electrotehnicii I - Conf. dr. Ing. Veronica PÂLTÂNEA, Facultatea de Inginerie Electrică
2.1. Introducere

în orice ramură a fizicii între mărimile care descriu starea unui


sistem și dinamica sa se stabilește un sistem de relații.
Unele relații au un caracter de mare generalitate fiind obținute
prin analiza, sistematizarea și extrapolarea unui mare număr de
experimente și reprezintă legile acelei ramuri a fizicii.
Legile au un caracter axiomatic (nu se demonstrează prin relații
matematice) și au un caracter necesar, în sensul că valabilitatea
lor poate fi evidențiată oriunde și oricând sunt respectate
ipotezele fizice conținute în enunțul lor.

Bazele Electrotehnicii I - Conf. dr. ing. Vetonica PÂLTÂNEA, Facultatea de Inginerie Electrică
în cele ce urmează se prezintă teoria macroscopicâ (fenomenologică) a câmpului
electromagnetic. Aceasta face abstracție de structura microscopică a sistemelor
fizice studiate considerând corpurile ca fiind medii continue.
Caracterul fenomenologic este datorat faptului că această teorie studiază
sistemele fizice numai din punctul de vedere al fenomenelor care se produc,
făcând apel direct la experiența (observarea fenomenului) pentru stabilirea
principalelor legi și introducerea mărimilor fizice cu care operează această teorie.
Fenomenele electromagnetice au fost cunoscute și studiate mult mai târziu decât
cele mecanice, termice sau optice ca urmare a inaccesibilității directe pentru
simțurile umane. Ca atare studiul lor a fost realizat indirect prin efectele asociate
câmpului electromagnetic.
Fundamentată prin lucrările lui J.C. Maxwell și H. Hertz în secolul al XIX -lea teoria
macroscopicâ a câmpului electromagnetic utilizează un aparat matematic mai
evoluat decât cel utilizat în studiul fenomenelor mecanice, termice și chimice.
Bazele Electrotehnicii I - Conf. dr. ing. Vetonica PÂLTÂNEA, Facultatea de Inginerie Electrică
2.2. Legile de material

2.2.1. Legea polarizației temporare


Enunț: In orice moment de timp și în orice punct din spațiu polarizația temporară P, este
funcție de intensitatea câmpului electric E

Pt=f(E)

Pentru cazul în care se consideră că fenomenele electrice se desfășoară în regim staționar


sau lent variabil în timp relația generală de definiție are unele forme particulare în funcție de
tipul (clasa) de materiale:
1. Clasa materialelor liniare, omogene și izotrope
2. Clasa materialelor neliniare, omogene și izotrope
3. Clasa materialelor anizotrope liniare sau neliniare

Bazele Electrotehnicii I - Conf. dr. Ing. Veronica PÂLTÂNEA, Facultatea de Inginerie Electrică 6
1. Clasa materialelor liniare, omogene și izotrope
Relația de dependență este de proporționalitate
d i rectă: ______________

unde e0 este permitivitatea absolută a vidului (mărime


constantă), iar Xe este susceptivitatea electrică
(constantă de material, pozitivă, adimensională)
2. Clasa materialelor neliniare, omogene și izotrope
Pentru materialele neliniare se consideră că unele
mărimi depind de alte mărimi de bază prin
intermediul unor relații matematice de tip neliniar.
Relația de dependență are forma:

Bazele Electrotehnicii I - Conf. dr. Ing. Veronica PÂLTÂNEA, Facultatea de Inginerie Electrică 7
3. Clasa materialelor anizotrope liniare sau neliniare
în cazul materialelor cu structură cristalină, care este uniformă și continuă în întreg volumul
acestora, proprietățile electrice se manifestă diferit, anizotrop, după diferite direcții preferențiale
din spațiu.
Relația de dependență are forma:
Xell Xel2 Xel3
Xe = Xe21 Xe22 Xe23
.Xe31 Xe32 Xe33.

unde xe este tensorul susceptivității electrice, iar 1 este direcția Ox, 2 direcția Oy și 3 direcția Oz.
Dacă Xeij are valoare constantă atunci materialul este anizotrop liniar, iar în cazul în care unul din
termenii tensorului are o dependență neliniară de tipul Xeij = Xeij(.Ek) atunci materialul este
anizotrop neliniar.

Bazele Electrotehnicii I - Conf. dr. Ing. Veronica PÂLTÂNEA, Facultatea de Inginerie Electrică 8
Vâscozitatea electrică (post-efect electric)
în cazul variației suficient de rapide în timp a intensității câmpului electric, polarizația temporară
instantanee urmărește cu o anumită întârziere (retardare) această variație.
Apare fenomenul de post-efect electric (vâscozitate electrică) și este însoțită de pierderi de
putere, degajate sub formă de căldură în procesul polarizării dielectricilor.

Bazele Electrotehnicii I - Conf. dr. ing. Veronica PÂLTÂNEA, Facultatea de Inginerie Electrică 9
2.2. Legile de material

2.2.2. Legea magnetizației temporare


Enunț: In orice moment de timp și în orice punct din spațiu magnetizația temporară Afr este
funcție de intensitatea câmpului magnetic#

Mt = f(H)

Pentru cazul în care se consideră că fenomenele magnetice se desfășoară în regim staționar


sau lent variabil în timp relația generală de definiție are unele forme particulare în funcție de
tipul (clasa) de materiale:
1. Clasa materialelor liniare, omogene și izotrope
2. Clasa materialelor neliniare, omogene și izotrope
3. Clasa materialelor anizotrope liniare sau neliniare

Bazele Electrotehnicii I - Conf. dr. Ing. Veronica PÂLTÂNEA, Facultatea de Inginerie Electrică 10
1. Clasa materialelor liniare, omogene și izotrope
Relația de dependență este de proporționalitate
directă:

undex?n este susceptivitatea magnetică (constantă de


material, pozitivă, adimensională)
2. Clasa materialelor neliniare, omogene și izotrope
Pentru materialele neliniare se consideră că unele
mărimi depind de alte mărimi de bază prin
intermediul unor relații matematice de tip neliniar.
Relația de dependență are forma:

Bazele Electrotehnicii I - Conf. dr. Ing. Veronica PÂLTÂNEA, Facultatea de Inginerie Electrică 11
3. Clasa materialelor anizotrope liniare sau neliniare
în cazul materialelor cu structură cristalină, care este uniformă și continuă în întreg volumul
acestora, proprietățile electrice se manifestă diferit, anizotrop, după diferite direcții preferențiale
din spațiu.
Relația de dependență are forma:

unde Xm este tensorul susceptivității magnetice, iar 1 este direcția Ox, 2 direcția Oy și 3 direcția Oz.
Dacă Xmij are valoare constantă atunci materialul este anizotrop liniar, iar în cazul în care unul din
termenii tensorului are o dependență neliniară de tipul xmij=xmij(Hk) atunci materialul este
anizotrop neliniar.

Bazele Electrotehnicii I - Conf. dr. Ing. Veronica PÂLTÂNEA, Facultatea de Inginerie Electrică 12
Vâscozitatea magnetică (post-efect magnetic)
în cazul variației suficient de rapide în timp a intensității câmpului magnetic, magnetizația
temporară instantanee urmărește cu o anumită întârziere (retardare) această variație.
Apare fenomenul de post-efect magnetic (vâscozitate magnetică) și este însoțită de pierderi de
putere, degajate sub formă de căldură în procesul magnetizării corpurilor.

Bazele Electrotehnicii I - Conf. dr. Ing. Veronica PÂLTÂNEA, Facultatea de Inginerie Electrică 13
2.2. Legile de material

2.2.3. Legea conducției electrice (Legea OHM)


Experiența arată că starea electrocinetică a conductoarelor apare ca urmare a existenței unui
câmp electric în interiorul lor.
Enunț: în orice moment de timp și în orice punct din spațiu densitatea curentului electric de
conducție Jeste funcție de intensitatea câmpului electric E

J = f(E)
In cazul acestei legi se consideră aceleași tipuri (clase) de materiale, dar se va dezvolta
analiza doar pentru primele două:
1. Clasa materialelor liniare, omogene și izotrope
2. Clasa materialelor neliniare, omogene și izotrope
3. Clasa materialelor anizotrope liniare sau neliniare
Bazele Electrotehnicii I - Conf. dr. ing. Vetonica PĂLTÂNEA, Facultatea de Inginerie Electrică 14
Forma locală a legii
în cazul materialelor omogene și izotrope (tipul 1 și 2), care pot avea atât un câmp electric propriu
E cât și un câmp electric imprimat E, în interiorul acestora, se definește forma locală a legii având
expresia:
E + Ei = pj
unde p este rezistivitate electrică (mărime scalară dependentă de natura materialului și de
temperatură).
Forma locală se poate exprima sub forma:
J = a(E + Ei)
unde o = - este conductivitate electrică.
p
Pentru conductoare perfect omogene d.p.d.v. structural, mecanic, termic și chimic, în careE, = 0,
forma locală a legii este:
E = pJ sau J = oE
Expresia generală a rezistivității electrice funcție de temperatură este:

Bazele Electrotehnicii I - Conf. dr. Ing. Vetonica PÂLTÂNEA, Facultatea de Inginerie Electrică 15
Forma integrală a legii
Prin integrarea formei locale se obține expresia globală a legii care se poate considera în cazul unei
porțiuni neramificate dintr-un un conductor electric definit între bornele A și B, de lungime 1,
caracterizat prin rezistivitatea p în interiorul căruia există câmpurile electrice E și £,

Bazele Electrotehnicii I - Conf. dr. Ing. Veronica PÂLTÂNEA, Facultatea de Inginerie Electrică 16
Forma integrală a legii

în cazul unui conductor filiform omogen, având lungimea 7, aria secțiunii transversalei pe întreaga
lungime și rezistivitatea p, se exprimă rezistența electrică a conductorului:

Unitatea de măsură a rezistenței electrice este ohm [fi].


Inversa rezistenței electrice se definește mărimea conductanță electrică G, având unitatea de
măsură siemens [S] sau ohm'1 [omega-1].
Cu aceste notații se obține:

Bazele Electrotehnicii I - Conf. dr. ing. Veronica PÂLTÂNEA, Facultatea de Inginerie Electrică 17
în teoria circuitelor electrice se poate identifica o reprezentare simbolică pentru cazul
conductorului filiform omogen, care conține o sursă de câmp electric imprimat.
Conform sensurilor de integrare de la A la B se obține configurația conformă cu regula de la
receptoare

Pentru calculul integralei de la B la A se identifică reprezentarea conform cu regula de la


generatoare

Bazele Electrotehnicii I - Conf. dr. Ing. Vetonica PÂLTÂNEA, Facultatea de Inginerie Electrică 18
în cazul în care câmpul electric imprimat este nul (E, = 0) în interiorul conductorului omogen având
rezistența electrică R se obține relația Ohm:

în cazul în care conductorul omogen se consideră perfect conductor electric (R = 0) se definește


Generatorul Ideal de Tensiune (G.I.T.) având două reprezentări simbolice
conform regulii de la generatoare conform regulii de la receptoare

Bazele Electrotehnicii I - Conf. dr. Ing. Veronica PÂLTÂNEA, Facultatea de Inginerie Electrică 19
OBSERVAȚII
în cazul în care câmpul electric imprimat este nul (E, = 0) forma locală a legii determină doua cazuri
limită:
• Cazul electroizolantului perfect (p -> oo) care implică un câmp electric finit

• Cazul conductorului perfect (o -> oo) care implică o densitate de curent finită

Bazele Electrotehnicii I - Conf. dr. Ing. Veronica PÂLTÂNEA, Facultatea de Inginerie Electrică 20
BAZELE
ELECTROTEHNICII I

Conf. Dr. Ing. Veronica PÂLTÂNEA

Bazele Electrotehnicii I - Conf. dr. ing. Veronica PÂLTÂNEA, Facultatea de Inginerie Electrică
Bazele Electrotehnicii I
Curs
Introducere. Mărimi electromagnetice
Legile teoriei macroscopice a electromagnetismului
Regimul electrostatic al câmpului electromagnetic
Regimul electrocinetic al câmpului electromagnetic
Regimul electrodinamic al câmpului electromagnetic
Seminar
Circuite electrice de curent continuu
Regimul electrostatic
Regimul electrodinamic

Bazele Electrotehnicii I - Conf. dr. ing. Veronica PÂLTÂNEA, Facultatea de Inginerie Electrică
Curs 2 Legile teoriei macroscopice a
electromagnetismului (Teoria Maxwell - Hertz)

2.1. Introducere 2.4. Legi generale


2.2. Legi de material 2.4.1. Legea fluxului electric
2.2.1. Legea polarizației temporare 2.4.2. Legea fluxului magnetic
2.4.3. Legea inducție electromagnetice
2.2.2. Legea magnetizației temporare
2.4.4. Legea circuitului magnetic
2.2.3. Legea conducției electrice
2.4.5. Legea conservării sarcinii electrice
2.3. Legi de legătură 2.4.6. Legea transformărilor energetice
2.3.1. Legea legăturii în câmp electric în conductoarele în stare electrocinetică
2.3.2. Legea legăturii în câmp magnetic 2.5. Legea electrolizei

Bazele Electrotehnicii I - Conf. dr. ing. Veronica PÂLTÂNEA, Facultatea de Inginerie Electrică
2.3. Legile de legătură

2.3.1. Legea legăturii în câmp electric


y - A
Enunț: In orice moment de timp, în orice punct din spațiu și în orice regim de funcționare
între inducția electrică D, intensitatea câmpului electric E și polarizațieP există relația

D = e0E+P
\_____________________________________ ________________________________________ y

Considerând relația de definiție a polarizației P = Pt + Pp și legea polarizației temporare


pentru materiale liniare, omogene și izotrope Pt = EoxeE se va obține:

D = e0E + P = e0E + Pt + Pp = e0E + zoxeE + Pp =


= Eo(l + + Pp = £0£r^ + Pp = £p + Pp

Bazele Electrotehnicii I - Conf. dr. ing. Veronica PÂLTÂNEA, Facultatea de Inginerie Electrică 4
Dielectric Material Relative Permittivity
Dry Air 1.0006
Se identifică două mărimi importante: Paper
PTFE, FEF (Teflon)
2.5 to 3.5
2.0
Polypropylene 2.20 to 2.28
1. Permitivitatea electrică relativă a materialului Polystyrene 2.4 to 3.2
Bakelite 3.5 to 6.0
Oak Wood 3.3
1 "F Xe Maple Wood 4.4
\_____________________________/ Birch Wood 5.2
Mărime adimensională, supraunitară, caracteristică Glass
Mica
3 to 10
5 to 7
Glass-Bonded Mica 6.3 to 9.3
unui material dat. Porcelain 6.5
Metal Oxide Powders 6 to 20
OBS.: https://www.engineersgarage.com/tutorials/articles-basic-electronics-
capacitors-capacitance-multiplate-capacitor-types-properties-of-
Permitivitatea electrică relativă este influențată de capacitor/

frecvența de variație a mărimilor electrice

2. Permitivitatea absolută a materialului

E = EqEj-
<_____________________________ J

Andryieuski, Andrei & Kuznetsova, Svetlana & Zhukovsky, Sergei &


Unitatea de măsură farad/metru [F/m] Kivshar, Yuri & Lavrinenko, Andrei. (2015). Water: Promising
Opportunities For Tunable All-dielectric Electromagnetic
Metamaterials. Scientific reports. 5. 13535. 10.1038/srepl3535.

Bazele Electrotehnicii I - Conf. dr. ing. Veronica PÂLTÂNEA, Facultatea de Inginerie Electrică
Cazul materialelor cu polarizație permanentă nulă
Dacă Pp = O atunci legea se poate exprima sub forma

D = eE

în acest caz liniile de câmp ale fluxului electric (caracterizat prin vectorul D) sunt coliniare cu
liniile de câmp electric (caracterizat prin vectorul E)

Această relație caracterizează comportarea materialelor liniare, omogene și izotrope

Bazele Electrotehnicii I - Conf. dr. ing. Veronica PĂLTÂNEA, Facultatea de Inginerie Electrică 6
Cazul materialelor cu histerezis electric - feroelectrice
Pentru o anumită clasă de materiale variația polarizației temporare în funcție de intensitatea
câmpului electric prezintă dependențe grafice diferite atunci când se aplică un câmp electric
crescător, respectiv descrescător.
Acest tip de dependență se numește ciclu de histerezis.
Se definește sub forma particulară Pt = f(E) sau, mai generală și utilizată D = f(E)

D- inducție electrică de saturație


5 / /

D, - inducție electrică remanentă


Ec-intensitatea câmpului electric coercitiv
Linia roșie reprezintă curba inițială
titanatul de bariu BaTiO3

Bazele Electrotehnicii I - Conf. dr. ing. Veronica PÂLTÂNEA, Facultatea de Inginerie Electrică 7
2.3. Legile de legătură

2.3.2. Legea legăturii în câmp magnetic


z - A
Enunț: In orice moment de timp, în orice punct din spațiu și în orice regim de funcționare
între inducția magnetică B, intensitatea câmpului magneticTZși magnetizațieM există relația

Considerând relația de definiție a polarizației M = Mt + Mp și legea magnetizației temporare


pentru materiale liniare, omogene și izotrope Mt — \mH se va obține:

Bazele Electrotehnicii I - Conf. dr. ing. Veronica PÂLTÂNEA, Facultatea de Inginerie Electrică
Magnetic Materials
Se identifică două mărimi importante: Material Relative Permeability //r
Vacuum 1

1. Permeabilitatea magnetică relativă a materialului Air


Water
1.0000004
0.999992
Copper 0.999994

Mr = 1 + Xm Aluminum
Silver
1.00002
0.9999S
V_______________________________________ / Nickel 600
Iron
Mărime adimensională, caracteristică unui material dat. Carbon Steel
5000
100
Transformer Steel 2000

OBS.: Mumetal
Supermalloy
50.000
1.000.000

Note: Values can often vary depending on purity and processing

Permeabilitatea magnetică relativă este influențată de


frecvența de variație a mărimilor magnetice

2. Permeabilitatea absolută a materialului


Z X

P = Ho Pr
\________________________________________

Unitatea de măsură henry/metru [H/m] flH/l

V. M. Paltanea, G. Paltanea and H. Gavrila, "Some important effects of the water jet
and laser cutting methods on the magnetic properties of the non-oriented silicon
iron sheets," 2015 9th International Symposium on Advanced Topics in Electrical
Engineering (ATEE), Bucharest, 2015, pp. 452-455, doi: 10.1109/ATEE.2015.7133856.

Bazele Electrotehnicii I - Conf. dr. ing. Veronica PÂLTÂNEA, Facultatea de Inginerie Electrică
Materiale magnetice
Liniare:
Diamagnetice: |ir = 0.9 ...,0.9999; < 0;xni G (IO-6... 10~4)
Carbon pirolitic,grafit, Bi, Ag, Hg, Pb, H2O
sunt respinse de câmpurile magnetice
se utilizează pentru levitație magnetică
Paramagnetice: |ir = 1.0009 ...,1.9999; xm > Oi Xm e (IO-6 ... 10-4)
Al, Cu, Au, Ti, Li, Mg, O2
sunt atrase de câmpurile magnetice intense

neliniare:
Feromagnetice: |ir = 500,..., 50000
aliaje pe bază de Fe, Ni sau Co
Ferimagnetice: ferite (Fe2O3) moi și dure
Antiferomagnetice: hematite, aliaje Cr, aliaje FeMn, aliaje NiO

Bazele Electrotehnicii I - Conf. dr. ing. Veronica PÂLTÂNEA, Facultatea de Inginerie Electrică 10
Cazul materialelor cu magnetizație permanentă nulă
Dacă Mp = O atunci legea se poate exprima sub forma

B = \iH

în acest caz liniile de câmp ale fluxului magnetic (caracterizat prin vectorul B) sunt coliniare cu
liniile de câmp magnetic (caracterizat prin vectorul H)

Această relație caracterizează comportarea materialelor liniare, omogene și izotrope

Bazele Electrotehnicii I - Conf. dr. ing. Veronica PÂLTÂNEA, Facultatea de Inginerie Electrică
Cazul materialelor cu histerezis magnetic -fero și feri magnetice
Pentru o anumită clasă de materiale variația magnetizației în funcție de intensitatea câmpului
magnetic prezintă dependențe grafice diferite atunci când se aplică un câmp magnetic crescător,
respectiv descrescător.
Acest tip de dependență se numește ciclu de histerezis.
Se definește sub forma particulară M = f(H) sau, mai generală și utilizată B = f(Jf)

Bs- inducție magnetică de saturație (Bmox)


Br- inducție magnetică remanentă
Hc-intensitatea câmpului magnetic coercitiv
Linia OABC reprezintă curba inițială

Bazele Electrotehnicii I - Conf. dr. ing. Veronica PÂLTÂNEA, Facultatea de Inginerie Electrică 12
Materialeferomagnetice
Magnetic moi:
Valori reduse ale intensității câmpului coercitiv
(0.5 A/m ... 150 A/m)
/ de saturație
Valori mari ale inducției /
Oțel electrotehnic (aliaj FeSi), aliaje FeNi, aliaje
FeCo
Magnetic semi-dure
Valori medii ale intensității câmpului coercitiv
(200 A/m ...20000 A/m)'
Materiale utilizate în înregistrări magnetice
(HDD, SSD)
Magnetic dure
Valori mari ale intensității câmpului coercitiv
(50 kA/m ... 500 kA/m) ’
/ de saturație
Valori medii ale inducției /
Magneți permanenți: NdFeB, SmCo, AINiCo,
Ferite dure
Bazele Electrotehnicii I - Conf. dr. ing. Veronica PÂLTÂNEA, Facultatea de Inginerie Electrică 13
2.4. Legi generale

2.4.1. Legea fluxului electric (Gauss's law)


---------------------------------------------------------------------------------------------------------------------------- X
s*

Enunț: în orice moment de timp fluxul electric care parcurge o suprafață închisă (E) este egal cu
sarcina electrică localizată în interiorul suprafeței închise (X):

^(s) = Q(de)

Relația arată că în cazul unor medii nepolarizate electric există o sursă a câmpului de vectori:
Liniile de câmp electric pornesc de pe corpurile încărcate pozitiv și ajung pe cele încărcate negativ
Liniile de câmp electric sunt curbe deschise
Corpurile încărcate cu sarcini electrice își pot asocia un câmp electric propriu

Bazele Electrotehnicii I - Conf. dr. ing. Veronica PÂLTÂNEA, Facultatea de Inginerie Electrică
Forma integrală a legii

Dacă se consideră o distribuție de volum a sarcini electrice forma integrală extinsă este:

Bazele Electrotehnicii I - Conf. dr. ing. Veronica PĂLTÂNEA, Facultatea de Inginerie Electrică 15
Forma locală a legii
Pentru a aduce ambele integrale la același domeniu de integrare se folosește teorema Gauss-
Ostrogradski

Distribuția liniilor de câmp, a vectorilor D și E pentru un corp punctual plasat în vid:


încărcat cu sarcina electrică pozitivă +q încărcat cu sarcina electrică negativă -q

Bazele Electrotehnicii I - Conf. dr. ing. Veronica PÂLTÂNEA, Facultatea de Inginerie Electrică
Cazul suprafețelor de discontinuitate
în cazul unor medii cu proprietăți electrice diferite parcurgerea acestora de către câmpul electric
este influențată de parametrii electrici ai materialelor respective.
între cele două medii electrice se găsește o suprafață de discontinuitate, care poate fi încărcată
cu o distribuție de suprafață a sarcinii electrice ps, astfel încât forma locală a legii se scrie:

Bazele Electrotehnicii I - Conf. dr. ing. Veronica PÂLTÂNEA, Facultatea de Inginerie Electrică
La trecerea câmpului electric printr-o suprafață de discontinuitate între două medii, încărcată cu
o sarcină electrică, componenta normală a inducției electrice are un salt de valoare egal cu
densitatea de suprafață a sarcinii electrice.

Dacă suprafața de discontinuitate nu este încărcată electric (ps = 0) atunci componenta


normală a vectorului inducție
/ electrică se conservă la trecerea dintr-un mediu în celălalt

^2n = Din

Bazele Electrotehnicii I - Conf. dr. ing. Veronica PÂLTÂNEA, Facultatea de Inginerie Electrică
2.4. Legi generale

2.4.2. Legea fluxului magnetic (Gauss's law for magnetism)


- --------------------------------------------------------------------------------------------------------------------------
Enunț: în orice moment de timp fluxul magnetic care parcurge o suprafață închisă (S) este nul:Φ
(S)

= 0

Relația arată:
t

Liniile de câmp magnetic sunt curbe închise


Liniile de câmp magnetic sunt curbe descrise în jurul sursei de câmp magnetic
Nu se pot identifica sarcinii magnetice, care să poată genera câmp magnetic propriu

Bazele Electrotehnicii I - Conf. dr. ing. Veronica PÂLTÂNEA, Facultatea de Inginerie Electrică 19
Forma integrală a legii

Forma locală a legii


Pentru a aduce ambele integrale la același domeniu de integrare se folosește teorema Gauss-
Ostrogradski

Bazele Electrotehnicii I - Conf. dr. ing. Veronica PÂLTÂNEA, Facultatea de Inginerie Electrică
Cazul suprafețelor de discontinuitate
în cazul unor medii cu proprietăți magnetice diferite, parcurgerea acestora de către câmpul
magnetic este influențată de parametrii magnetici ai materialelor respective.
între cele două medii magnetice se găsește o suprafață de discontinuitate, astfel încât forma
locală a legii se scrie:

Lî”1

Bazele Electrotehnicii I - Conf. dr. ing. Veronica PÂLTÂNEA, Facultatea de Inginerie Electrică
La traversarea câmpului magnetic printr-o suprafață de discontinuitate între două medii
componenta normală a inducției magnetice se conservă.

Bazele Electrotehnicii I - Conf. dr. ing. Veronica PÂLTÂNEA, Facultatea de Inginerie Electrică
BAZELE
ELECTROTEHNICII I

Conf. Dr. Ing. Veronica PÂLTÂNEA

Bazele Electrotehnicii I - Conf. dr. ing. Veronica PÂLTÂNEA, Facultatea de Inginerie Electrică
Bazele Electrotehnicii I
Curs
Introducere. Mărimi electromagnetice
Legile teoriei macroscopice a electromagnetismului
Regimul electrostatic al câmpului electromagnetic
Regimul electrocinetic al câmpului electromagnetic
Regimul electrodinamic al câmpului electromagnetic
Seminar
Circuite electrice de curent continuu
Regimul electrostatic
Regimul electrodinamic

Bazele Electrotehnicii I - Conf. dr. ing. Veronica PÂLTÂNEA, Facultatea de Inginerie Electrică
Curs 2 Legile teoriei macroscopice a
electromagnetismului (Teoria Maxwell - Hertz)

2.1. Introducere 2.4. Legi generale


2.2. Legi de material 2.4.1. Legea fluxului electric
2.2.1. Legea polarizației temporare 2.4.2. Legea fluxului magnetic
2.4.3. Legea inducție electromagnetice
2.2.2. Legea magnetizației temporare
2.4.4. Legea circuitului magnetic
2.2.3. Legea conducției electrice
2.4.5. Legea conservării sarcinii electrice
2.3. Legi de legătură 2.4.6. Legea transformărilor energetice
2.3.1. Legea legăturii în câmp electric în conductoarele în stare electrocinetică
2.3.2. Legea legăturii în câmp magnetic 2.5. Legea electrolizei

Bazele Electrotehnicii I - Conf. dr. ing. Veronica PĂLTÂNEA, Facultatea de Inginerie Electrică
2.4. Legi generale

2.4.3. Legea inducției electromagnetice (Faraday's law)


Enunț: Tensiunea electromotoare indusă în lungul unei curbe închise (f) este egală cu viteza de
scădere în timp a fluxului magnetic 4»(-Sr) care parcurge orice suprafață deschisă (Sr) mărginită de
curba închisă (T):

L______
Experimental se constată că la variația în timp a câmpului magnetic se produce
câmp electric. Fenomenul, constând în apariția în lungul unui contur închis a
unei tensiuni electromotoare, poartă numele de inducție electromagnetică și a
fost pus în evidență prin experiențele lui Faraday, în 1831. Ca efect al acestui
fenomen, în circuitele electrice închise apar curenți induși cu un astfel de sens,
încât se opun cauzei care i-a produs (regula lui Lenz).

Bazele Electrotehnicii I - Conf. dr. ing. Veronica PÂLTÂNEA, Facultatea de Inginerie Electrică 4
Formele integrale ale legii
în cazul mediilor imobile expresia formei integrale este:

Considerând mediul în care se desfășoară fenomenele electromagnetice ca fiind în mișcare cu o


viteză v atunci curba (T) fiind atașată corpului de probă se va deplasa cu mediul respectiv.
Expresia dezvoltată a legii pentru corpuri în mișcare este (se aplică derivata substanțială de flux):

Bazele Electrotehnicii I - Conf. dr. ing. Veronica PÂLTÂNEA, Facultatea de Inginerie Electrică 5
Se identifică doi termenifoarte importanți:

• Tensiunea electromotoare de transformare

Este caracteristică modului de obținere a tensiunii electrice în cazul transformatoarelor electrice


și necesită prezența unui câmp magnetic variabil în timp

• Tensiunea electromotoare de mișcare


9

Este caracteristică modului de obținere a tensiunii electrice în cazul generatoarelor electrice și


necesită un câmp magnetic constant care se deplasează cu o viteză oarecare

Bazele Electrotehnicii I - Conf. dr. ing. Veronica PÂLTÂNEA, Facultatea de Inginerie Electrică
Formele locale ale legii
Pentru a aduce ambele integrale la același domeniu de integrare se folosește teorema Stokes
în cazul mediilor imobile expresia formei locale (ecuația a doua Maxwell) este:

Expresia pentru corpuri în mișcare este:

Bazele Electrotehnicii I - Conf. dr. ing. Veronica PÂLTÂNEA, Facultatea de Inginerie Electrică 7
Cazul suprafețelor de discontinuitate
în cazul unui punct M aflat la limita de separație între două medii se consideră o curbă închisă
(O definită imediat în jurul acestui punct, ceea ce permite să considerăm aria suprafeței (Sr)
aproximativ nulă. în aceste condiții legea de exprimă:

Rezultă că traversarea unei suprafețe de discontinuitate de către liniile de camp electric se face
întotdeauna prin conservarea componentei tangențiale a intensității câmpului electric.

Bazele Electrotehnicii I - Conf. dr. ing. Veronica PÂLTÂNEA, Facultatea de Inginerie Electrică 8
Observație:
9
în cazul regimului de funcționare staționar sau cvasistaționar, dacă curbă închisă (T) de definiție
a legii, descrie un circuit electric, care se găsește într-un câmp magnetic nul sau constant
(derivata fluxului magnetic fiind astfel nulă) atunci legea inducției electromagnetice se scrie:

Considerând că este posibilă descompunerea curbei (T) în n curbe deschise (Q), (k = 1 n)


care descriu laturile circuitului electric atunci relația
f de mai sus este:

Pentru că în cazul integralelor se ține cont de sensul de integrare în comparație cu sensul curbei
închise, relația obținută se poate exprima sub forma Teoremei Kirchhoff II
<(A)
/ uk = 0

Bazele Electrotehnicii I - Conf. dr. ing. Veronica PÂLTÂNEA, Facultatea de Inginerie Electrică 9
2.4. Legi generale

2.4.4.Legea circuitului magnetic (Ampere-Maxwell's law)


Enunț: Tensiunea magnetomotoare u„nn(r) produsă în lungul unei curbe închise (T) este egală cu
suma dintre intensitatea curentului electric de conducție, care parcurge orice suprafață deschisă (Sr)
mărginită de curba închisă (T) și viteza de creștere în timp a fluxului electric ) prin aceeași
suprafață:

Experiența arată că există câmp magnetic asociat atât corpurilor magnetizate, cât și corpurilor
electrizate (încărcate sau polarizate electric) aflate în mișcare și respectiv a corpurilor parcurse de
curenți electrici de conducție. Câmp magnetic se mai poate obține prin variația suficient de rapidă în
timp a unui câmp electric.
Bazele Electrotehnicii I - Conf. dr. ing. Veronica PÂLTÂNEA, Facultatea de Inginerie Electrică 10
Forme particulare ale curentului electric de conducție:
Se pot întâlnii diferite metode prin care un curent electric care parcurge un conductor electric
poate genera câmp magnetic. Astfel se definesc:
curent electric de conducție care parcurge un conductor electric filiform

curent electric de conducție care parcurge un conductor masiv

solenația sau curentul electric de conducție care parcurge un număr N de spire

Termenul al doilea al relației de definiție se numește curent hertzian


/ t 9

Bazele Electrotehnicii I - Conf. dr. ing. Veronica PÂLTÂNEA, Facultatea de Inginerie Electrică 11
Formele integrale ale legii
în cazul mediilor imobile expresia formei integrale este:

Considerând mediul în care se desfășoară fenomenele electromagnetice ca fiind în mișcare cu o


viteză v atunci curba (T) fiind atașată corpului de probă se va deplasa cu mediul respectiv.
Expresia dezvoltată a legii pentru corpuri în mișcare este (se aplică derivata substanțială de flux):

Bazele Electrotehnicii I - Conf. dr. ing. Veronica PÂLTÂNEA, Facultatea de Inginerie Electrică
Curent electric de conductie
/

Curent de deplasare

Curent de convectie
/

Curent Rontgen

Curentul Rontgen - nu a putut fi măsurat experimental în forma dată de relația matematică,


dar în schimb s-a verificat experimental că în locul inducției electrice D ar trebui să existe
polarizația P. Această neconcordanță între teorie și practica experimentală nu prezintă însă
importanță practică, astfel încât se poate ignora. Relația este demonstrată teoretic pe baza
teoriei relativității.

Bazele Electrotehnicii I - Conf. dr. ing. Veronica PĂLTÂNEA, Facultatea de Inginerie Electrică 13
Formele locale ale legii
Pentru a aduce ambele integrale la același domeniu de integrare se folosește teorema Stokes
în cazul mediilor imobile expresia formei locale (prima ecuație Maxwell) este:

Expresia pentru corpuri în mișcare este:

Bazele Electrotehnicii I - Conf. dr. ing. Veronica PÂLTÂNEA, Facultatea de Inginerie Electrică 14
Cazul suprafețelor de discontinuitate
în cazul unui punct M aflat la limita de separație între două medii, între care nu există un curent
de suprafață, se consideră o curbă închisă (T) definită imediat în jurul acestui punct, ceea ce
permite să considerăm aria suprafeței (Sr) aproximativ nulă. în aceste condiții legea de exprimă:

Rezultă că traversarea unei suprafețe de discontinuitate, care nu conține un curent de suprafață,


de către liniile de câmp magnetic se face întotdeauna prin conservarea componentei tangențiale
a intensității câmpului magnetic.
Bazele Electrotehnicii I - Conf. dr. ing. Veronica PÂLTÂNEA, Facultatea de Inginerie Electrică 15
Dacă la limita de separație dintre cele două medii există un curent de suprafață ]s, atunci
traversarea liniilor de câmp magnetic a suprafeței de separație se realizează cu un salt al
componentei tangențiale a intensității câmpului magnetic:

Bazele Electrotehnicii I - Conf. dr. ing. Veronica PÂLTÂNEA, Facultatea de Inginerie Electrică
BAZELE
ELECTROTEHNICII I

Conf. Dr. Ing. Veronica PÂLTÂNEA

Bazele Electrotehnicii I - Conf. dr. ing. Veronica PÂLTÂNEA, Facultatea de Inginerie Electrică
Bazele Electrotehnicii I
Curs
Introducere. Mărimi electromagnetice
Legile teoriei macroscopice a electromagnetismului
Regimul electrostatic al câmpului electromagnetic
Regimul electrocinetic al câmpului electromagnetic
Regimul electrodinamic al câmpului electromagnetic
Seminar
Circuite electrice de curent continuu
Regimul electrostatic
Regimul electrodinamic

Bazele Electrotehnicii I - Conf. dr. ing. Veronica PÂLTÂNEA, Facultatea de Inginerie Electrică
Curs 2 Legile teoriei macroscopice a
electromagnetismului (Teoria Maxwell - Hertz)

2.1. Introducere 2.4. Legi generale


2.2. Legi de material 2.4.1. Legea fluxului electric
2.2.1. Legea polarizației temporare 2.4.2. Legea fluxului magnetic
2.4.3. Legea inducție electromagnetice
2.2.2. Legea magnetizației temporare
2.4.4. Legea circuitului magnetic
2.2.3. Legea conducției electrice
2.4.5. Legea conservării sarcinii electrice
2.3. Legi de legătură 2.4.6. Legea transformărilor energetice
2.3.1. Legea legăturii în câmp electric în conductoarele în stare electrocinetică
2.3.2. Legea legăturii în câmp magnetic 2.5. Legea electrolizei

Bazele Electrotehnicii I - Conf. dr. ing. Veronica PÂLTÂNEA, Facultatea de Inginerie Electrică
2.4. Legi generale

2.4.5. Legea conservării sarcinii electrice


Enunț: Intensitatea curentului electric de conducție care traversează o suprafață închisă (£) este
egală cu viteza de scădere în timp a sarcinii electrice conținute în domeniul (Z)£) mărginit de
suprafața închisă:

Experiența arată că atunci când sarcina electrică dintr-un domeniu spațial variază, suprafața închisă,
care delimitează domeniul, este străbătută de un curent de conducție.

Bazele Electrotehnicii I - Conf. dr. ing. Veronica PĂLTÂNEA, Facultatea de Inginerie Electrică
Formele integrale ale legii
Dacă se consideră o distribuție de volum a sarcini electrice, iar curentul electric de conducție este
caracterizat local printr-o densitate de suprafață, atunci forma integrală, în cazul mediilor
imobile, are expresia:
7
Jj’d,! = - A J p„diz

(S)(Pș) y
Considerând mediul în care se desfășoară fenomenele electromagnetice ca fiind în mișcare cu o
viteză v atunci suprafața (Z) fiind atașată corpului de probă se va deplasa cu mediul respectiv.
Expresia dezvoltată a legii pentru corpuri în mișcare este (se aplică derivata substanțială de flux):

Bazele Electrotehnicii I - Conf. dr. ing. Veronica PÂLTÂNEA, Facultatea de Inginerie Electrică 5
Curent electric de conductie
/ Curent electric de convectie t

Curentul electric de convecție corespunde deplasării în raport cu un sistem de referință a


sarcinilor electrice atașate corpurilor.
Relația arată că sarcina electrică dintr-un domeniu delimitat de (Z) scade atât datorită curentului
de conducție cât și a celui de convecție care părăsesc suprafața (X).

Bazele Electrotehnicii I - Conf. dr. ing. Veronica PÂLTÂNEA, Facultatea de Inginerie Electrică
Formele locale ale legii
Pentru a aduce ambele integrale la același domeniu de integrare se folosește teorema G-0
în cazul mediilor imobile expresia formei locale este:

Expresia pentru corpuri în mișcare este:

Bazele Electrotehnicii I - Conf. dr. ing. Veronica PÂLTÂNEA, Facultatea de Inginerie Electrică
Cazul suprafețelor de discontinuitate
în cazul unui punct M aflat la limita de separație între două medii, între care există o suprafață
încărcată cu o densitate a sarcinii electrice ps, se definește imediat în jurul acestui punct o
suprafață închisă (£). în aceste condiții legea de exprimă:

Bazele Electrotehnicii I - Conf. dr. ing. Veronica PÂLTÂNEA, Facultatea de Inginerie Electrică 8
La trecerea liniilor de curent printr-o suprafață de discontinuitate între două medii, încărcată cu
sarcină electrică, componenta normală a densității curentului electric de conducție are un salt de
valoare egal cu viteza de scădere în timp a densității de suprafață a sarcinii electrice.

Dacă suprafața de discontinuitate nu este încărcată electric (ps = 0) atunci componenta


normală a vectorului densității curentului electric de conducție se conservă la trecerea dintr-un
t t

mediu în celălalt

Bazele Electrotehnicii I - Conf. dr. ing. Veronica PÂLTÂNEA, Facultatea de Inginerie Electrică
Observație:
în cazul regimului de funcționare staționar sau cvasistaționar, dacă suprafața închisă (E) de
definiție a legii, descrie un nod de circuit electric, astfel încât în interiorul acesteia sarcina electrică
este permanent nulă sau constantă în timp (derivata sarcinii electrice fiind astfel nulă) atunci legea
conservării sarcinii electrice se scrie:

Considerând că transferul de curent electric de conductie se


realizează doar prin intermediul a n conductoare electrice având
aria secțiunii transversale (Sfe), (k=1 % n), care intersectează
suprafața închisă (X) atunci se poate identifica o suprafață (S)
astfel încât E = S U Relația de mai sus devine:

Bazele Electrotehnicii I - Conf. dr. ing. Veronica PÂLTÂNEA, Facultatea de Inginerie Electrică
Pentru că în cazul integralelor se ține cont de sensul de integrare în comparație cu sensul
normalei la suprafața închisă, relația obținută se poate exprima sub forma Teoremei Kirchhoff I

Bazele Electrotehnicii I - Conf. dr. ing. Veronica PÂLTÂNEA, Facultatea de Inginerie Electrică
2.4. Legi generale

2.4.6. Legea transformărilor energetice în conductoarele în stare


electrocinetică (Joule's law)
/-------------------------------------------------------------------------------------------------------------------------------------------------------------------------------------------------------------------------------------------------------------------------------------

Enunț: Densitatea de volum a puterii pe care câmpul electromagnetic o cedează unui conductor aflat
în stare electrocinetică este egală cu produsul scalar dintre intensitatea câmpului electric și
densitatea curentului electric de conductie stabilit în conductor:

.P! = EJ
Este o lege generală de formă locală care sintetizează observațiile experimentale potrivit cărora:
orice conductor parcurs de curent electric de conducție degajă căldură;
dacă în plus conductorul este sediul unui câmp electric imprimat, între sursele acestui câmp și
curentul electric de conducție are loc un schimb energetic reversibil

Bazele Electrotehnicii I - Conf. dr. ing. Veronica PÂLTÂNEA, Facultatea de Inginerie Electrică 12
Considerând legea conducției electrice sub forma:

E + Et = pj => E = pj - Ei,

această lege se poate dezvolta astfel:

Pj = EJ = (p/ - E()/ = pj2 - EJ = pR~pe

unde cei doi termeni sunt:


Pr = pJ2 > 0 densitatea de volum a puterii transmise de câmpul electromagnetic
conductorului și transformată ireversibil în căldură (efect Joule-Lenz)
pe = EiJ <0 densitatea de volum a puterii schimbate reversibil de câmpul electromagnetic
cu sursa câmpului electric imprimat

• Dacă <(Ei,J) < - atunci pe > 0 și puterea este transmisă de sursa de câmp
2
electric imprimat către câmpul electromagnetic

• Dacă <(Ej,J) > | atunci pe < 0 și puterea este transmisă de câmpul


electromagnetic către sursa de câmp electric imprimat

Bazele Electrotehnicii I - Conf. dr. ing. Veronica PÂLTÂNEA, Facultatea de Inginerie Electrică 13
Forma integrală a legii
Forma integrală a legii se obține prin integrarea densității de putere pe volumul unui conductor
filiform liniar, omogen și izotrop ceea ce presupune că toți vectorii sunt paraleli:

PR = Ri2 > 0 reprezintă puterea consumată (disipată) în conductor sub formă de căldură
Pe = eti' 0 reprezintă puterea generată/consumată de sursa de câmp electric imprimat
în teoria circuitelor se consideră circuite electrice care nu interacționează cu surse de câmp
electromagnetic exterior acestora astfel încât Pj = 0
Bazele Electrotehnicii I - Conf. dr. ing. Veronica PÂLTÂNEA, Facultatea de Inginerie Electrică 14
Puterea totală cedată de câmpul electromagnetic unei porțiuni de conductor filiform în procesul de conducție
electrică este egală cu produsul dintre tensiunea electrică în lungul firului (la borne) și intensitatea curentului
electric care parcurge conductorul.

Mărimea electrică P reprezintă puterea activă și are unitatea de măsura watt [W].

Integrala de timp a puterii se numește energie și are unitatea de măsură joule [J] sau watt-secundă [Ws]

W= I Pdt

în energetică energia electrică se măsoară în kilowattoră [kWh]. Relația dintre diferitele unități de măsură este:
l[kWh] = 3,6 x 106[Ws] 860 [kcal]

Bazele Electrotehnicii I - Conf. dr. ing. Veronica PÂLTÂNEA, Facultatea de Inginerie Electrică 15
2.5. Legea electrolizei

Enunț: Masa de substanță depusă în unitatea de timp la unul din electrozii unei celule electrolitice
este egală cu produsul dintre intensitatea curentului electric de conducție care parcurge celula de

electroliză și raportul dintre echivalentul electrochimie — și constanta universală Faraday Fo:

Constanta universală Faraday Fo = 96490 [C]

Această lege caracterizează electroliții (conductoare de speța a doua în care trecerea curentului
electric de conducție este însoțită de reacții chimice)

Bazele Electrotehnicii I - Conf. dr. ing. Veronica PÂLTÂNEA, Facultatea de Inginerie Electrică 16
BAZELE
ELECTROTEHNICII I

Conf. Dr. Ing. Veronica PĂLTÂNEA

Bazele Electrotehnicii I - Conf. dr. ing. Veronica PĂLTÂNEA, Facultatea de Inginerie Electrică
Bazele Electrotehnicii I
Curs
Introducere. Mărimi electromagnetice
Legile teoriei macroscopice a electromagnetismului
Regimul electrostatic al câmpului electromagnetic
Regimul electrocinetic al câmpului electromagnetic
Regimul electrodinamic al câmpului electromagnetic
Seminar
Circuite electrice de curent continuu
Regimul electrostatic
Regimul electrodinamic

Bazele Electrotehnicii I - Conf. dr. ing. Veronica PĂLTÂNEA, Facultatea de Inginerie Electrică
Curs 3 Regimul electrostatic al
câmpului electromagnetic
3.1. Relații fundamentale în regim electrostatic
Regimul electrostatic: mărimile fizice nu variază în timp și nu există fenomene
termodinamice (nu există schimburi și transformări de energie).
Legea polarizației temporare pt = zoxeE

Legea legăturii în câmp electric d = eoerE + pp = zE + pp


Legea conservării sarcinii electrice

" dT în regim electrostatic nu există j K ^(Ds) = ^Jclk = constant


i(s) - 0
curent electric de conductie
= constant

Bazele Electrotehnicii I - Conf. dr. ing. Veronica PĂLTÂNEA, Facultatea de Inginerie Electrică
Legea fluxului electric (Teorema Coulomb)

^(S) - <7(ds) => 1 DdA = Q( dz)

d)
<7(D2) - dq = pvdV
j(D£) J(DS)

f dq = f Psd/1
Q(sr) =
J(sr) J(sr)

<7(C) = dq = p(dl
J(C) J(C)

Bazele Electrotehnicii I - Conf. dr. ing. Veronica PĂLTÂNEA, Facultatea de Inginerie Electrică
Legea inducției electromagnetice (Teorema potențialului electrostatic)

= constant sau zero

Relația arată că circulația intensității câmpului electrostatic este nulă pe orice curbă închisă.
Consecințe:
r

■ în câmp electrostatic nu există linii de câmp închise;


■ Tensiunea electrică între două puncte nu depinde de drum
(de curba de integrare sau de traseul dintre cele două puncte)

UC1 — UC2 — UAB

Bazele Electrotehnicii I - Conf. dr. ing. Veronica PĂLTÂNEA, Facultatea de Inginerie Electrică
■ Se definește o mărime scalară care se poate asocia unui punct în spațiu numită potențial electric
• Se notează cu V și are unitatea de măsură volt [V]
• Relația • dV = —Edl
• de definiție:
• Pentru o curbă deschisă care descrie un traseu între un punct curent (de calcul) P și un punct fix (de
referință) Po se aplică teorema potențialului electrostatic

• Relația finală reprezintă forma locală a teoremei potențialului electrostatic

■ Tensiunea electrică între două puncte din câmp electrostatic poate fi exprimată ca diferența
potențialelor celor două puncte (se pot considera punctele A (punct curent) și B (punct fix))

V(A) - V(B) = uAB

Bazele Electrotehnicii I - Conf. dr. ing. Veronica PĂLTÂNEA, Facultatea de Inginerie Electrică
Legea conducției electrice
Deoarece curentul electric de conducție este nul, atunci densitatea curentului electric de
conductie este de asemenea nulă
7=0
în aceste condiții forma locală a L.C.EI. are expresia:
E + £f = 0
care reprezintă condiția de echilibru electrostatic
Pentru conductoare omogene fără câmp electric imprimat(£; = 0)în orice punct al acestora E = 0
Consecință:
r

■ La introducerea într-un câmp electric, un conductor neutru se electrizează. Fenomenul, denumit


electrizare prin influență, constă în repartizarea unor sarcini electrice pe suprafața conductorului,
fără modificarea sarcinii sale totale (nule în cazul conductoarelor neutre), astfel încât câmpul
electric determinat de această repartiție de sarcină să anuleze câmpul exterior în toate punctele
din interiorul conductorului pentru a satisface condiția de echilibru electrostatic.

Bazele Electrotehnicii I - Conf. dr. ing. Veronica PĂLTÂNEA, Facultatea de Inginerie Electrică
Proprietăți ale conductoarelor electrice omogene fără câmp electric imprimat:

■ Intensitatea câmpului electrostatic în interiorul acestor conductoare este nulă.


■ Toate punctele din interiorul unui conductor au același potențial electric (sau tensiunea electrică în
lungul unui conductor electric omogen este nulă)
• Dacă se consideră că P și Po sunt două puncte de pe conductorul electric atunci:
p
rn = r(p0) - J Edl => V(/>) = V(Ă>„) - 0=>K(P) = V(P0)
E=0
Po
■ Sarcina electrică a conductoarelor este repartizată pe suprafața exterioară, iar sarcina din interiorul
conductoarelor este nulă.

Bazele Electrotehnicii I - Conf. dr. ing. Veronica PĂLTÂNEA, Facultatea de Inginerie Electrică
■ Inducția electrică este normală pe suprafața acestor conductoare și numeric egală în orice punct cu
densitatea superficială a sarcinii electrice.

^2n ~ Ps

■ în cavitățile fără sarcini electrice din interiorul conductoarelor, câmpul electric este nul (efect
Faraday). Acest efect se folosește în instalațiile de î.t. pentru ecranarea (prin conductoare legate la
pământ) a locurilor de observație în care se află personal operator.

Bazele Electrotehnicii I - Conf. dr. ing. Veronica PĂLTÂNEA, Facultatea de Inginerie Electrică
3.2. Tub de flux electric
Se numește tub de flux electric porțiunea de câmp delimitată de totalitatea liniilor de câmp
care trec prin toate punctele unei curbe închise.

^(S) - Q(ds)
Z = Si U S2 u st
n(si) 1 D

Dn(S;)cL4 = ^ - vp2 =

Dacă în interiorul suprafeței închise nu există sarcini electrice atunci fluxul electric se
conservă
^1 = ^2
Bazele Electrotehnicii I - Conf. dr. ing. Veronica PĂLTÂNEA, Facultatea de Inginerie Electrică 10
3.3. Teorema refracției liniilor de câmp electric
La trecerea printr-o suprafață de discontinuitate între două medii liniare și izotrope, cu
permitivități electrice diferite e1z respectiv e2, liniile de câmp electric se refractă.
Calculând raportul dintre tangenta unghiului de incidență ax și a unghiului de refracție a2 se
obține:
/
Dlt
tg«l _ Dm _ Dlt _ £lElt £1
tga2 D2ț ^2^2t £2
^2n

L. leg. în câmp el. => D = zE


L. F. El.=> Dln = D2n
L. Ind. Elmag. => Eit — ^2t

Relația arată că la trecerea dintr-un material cu permitivitate electrică mai mareîntr-unul cu


permitivitate electrică mai mică (ex » e2), liniile de câmp electric se apropie de normală
(a2-»0).

Bazele Electrotehnicii I - Conf. dr. ing. Veronica PĂLTÂNEA, Facultatea de Inginerie Electrică
3.4. Calcului câmpurilor coulombiene cu ajutorul legii fluxului electric

e£cL4 =
r (s)

Aplicația 1
Un corp punctual încărcat cu sarcina pozitivă +Q se află într-un mediu liniar, izotrop, infinit extins
de permitivitate e. Să se determine intensitatea câmpului electric E într-un punct M având vectorul
de poziție r.
în jurul unui corp punctual se considera o suprafață închisă (Z) de forma unei sfere de rază r, care
este cea mai apropiată geometrie de forma acestuia. Se consideră punctul M plasat pe suprafața
acestei sfere caracterizat prin vectorul de poziție r.

sFdzl = J sfin^d/l = J e|£"||n21 cos(f, nz)d/l = J S.E • 1 • cos(0°) d4 =


(S) (S) (S)

7 Q Q r
4Trr2tE = Q^E = —^z^E = -—^-zz-
47rerz 47TErz r
Bazele Electrotehnicii I - Conf. dr. ing. Veronica PĂLTÂNEA, Facultatea de Inginerie Electrică 12
Pentru structuri de corpuri în care sarcina electrică ±Q este distribuită sub forma unor densități de sarcină
electrică dq (q dq) și se va considera un câmp electric elementar caracterizat prin d£ (jE df).
Pe baza relației identificate anterior se scrie:
1 dq 1 dqr
dE =------- ~=>dE =------- ~~
47T£r2 47T£F2r

în cazul densități de volum a sarcinii electrice dq = pvdV

(o)

Bazele Electrotehnicii I - Conf. dr. ing. Veronica PĂLTÂNEA, Facultatea de Inginerie Electrică
în cazul densități de suprafață a sarcinii electrice dq = pscL4

Bazele Electrotehnicii I - Conf. dr. ing. Veronica PĂLTÂNEA, Facultatea de Inginerie Electrică
în cazul densități de linie a sarcinii electrice dq = Pfdl

Bazele Electrotehnicii I - Conf. dr. ing. Veronica PĂLTÂNEA, Facultatea de Inginerie Electrică
Aplicația 2
Un conductor filiform, rectiliniu, încărcat cu densitatea de linie pz > 0 se află plasat în vid. Să se
determine intensitatea câmpului electric E într-un punct M aflat la distanța a de conductor.
Capetele conductorului formează cu punctul de calcul unghiurile ctj și a2.

(c)

l = a ctg a
a
r = —
sin a

OBSERVAȚIE: Deoarece pz > 0 (încărcarea electrostatică a conductorului este cu o sarcină electrică


pozitivă) atunci sensul vectorului intensității câmpului electric E este spre exterior (pleacă de pe
conductorul încărcat pozitiv). Dacă pz < 0 atunci sensul vectorului intensității câmpului electric E
este orientat spre conductor.
Bazele Electrotehnicii I - Conf. dr. ing. Veronica PĂLTÂNEA, Facultatea de Inginerie Electrică 16
a2 «2
Pl ~Pi sinar2
f si-- -pz Pi
= da = - ------- sin a da = -(cos a! + cos a2)
4ns 0 47T£o J r2 a 47ra£0 J 4nas o
Tr-ar n-a±

a2 «2
r. _ Pl f cos ar2 — [ Pi
da = cos a da = -------- (sina^ — sina7)
47T£O J r2 a 4nas0 J 47TO£O
n-a1

E = Eni - Et j

———J (cos ai + cos a?)2 + (sin a-, — sin a?)2 = y/2 + 2 cos a! cos a2 — 2 sin a! sin a2
47ra£0 4nas0
V2[l twsU + a2)]
4 ^2 + cos(a1 + a2) + cos(a1 — a2) — cos^ — a2) + cos(at + a2) =

2 = Pt Pl ?«! + a2\

4nas0 J
2•2• )] 2nas 27ra£0 C°S \ 2 /

Bazele Electrotehnicii I - Conf. dr. ing. Veronica PĂLTÂNEA, Facultatea de Inginerie Electrică 17
Aplicația 3
Un conductor filiform, rectiliniu, cu forma geometrică din figură, este încărcat cu densitatea de linie
pz fiind plasat în vid. Să se determine intensitatea câmpului electric E în punctul M.
Se cunosc: a, l, = a4 = -, a2 = -, a3 = -.

Problema se rezolvă prin metoda superpoziției (a adunării efectelor) considerând geometria


formată din două segmente, vertical și orizontal. Fiecare segment va determina câte o componentă
a vectorului rezultant al intensității câmpului electric în punctul M.

Bazele Electrotehnicii I - Conf. dr. ing. Veronica PĂLTÂNEA, Facultatea de Inginerie Electrică 18
Pentru porțiunea de conductor AB

Pz V3-1
2'rTa£0 2V2

P;
8iTa£0

Bazele Electrotehnicii I - Conf. dr. ing. Veronica PĂLTÂNEA, Facultatea de Inginerie Electrică
Pl Pi (( cos_
— n +cos_)
(cos a3 + cos a4) =
4ttZ£0
Pl
8tt/c0

20
Se aplică metoda superpoziției. Se atașează un sistem de axe Oxy în punctul M și se realizează
suma algebrică a tuturor componentelor care se regăsesc după axa Ox, respectiv Oy.
n—

£ab/'

ab» bcc 8ira£0 8ttZ£0

Pz
ABt + ^BCn - 4kc?£o
81T£0

Bazele Electrotehnicii I - Conf. dr. ing. Veronica PĂLTÂNEA, Facultatea de Inginerie Electrică
Exemple
Un conductor electric filiform, rectiliniu, format din două segmente AB (pz> 0), respectiv BC (pz< 0)
se află plasat în vid. Să se determine intensitatea câmpului electric E într-un punct M aflat la distanța
a de conductor.
Se cunosc: a, = a4 = p a2 = a3 = “■

Bazele Electrotehnicii I - Conf. dr. ing. Veronica PĂLTÂNEA, Facultatea de Inginerie Electrică 22
Exemple
Un conductor electric filiform, rectiliniu, format din două segmente AB (pz> 0), respectiv BC (pz> 0)
se află plasat în vid. Să se determine intensitatea câmpului electric E într-un punct M aflat la distanța
a de conductor.
Se cunosc: a, = tt, a2 =

Bazele Electrotehnicii I - Conf. dr. ing. Veronica PĂLTÂNEA, Facultatea de Inginerie Electrică
Aplicația 4
Un conductor electric filiform, având forma unei spire circulare de rază a, încărcat cu densitatea de
linie pz > 0 se află plasat în vid. Să se determine intensitatea câmpului electric E într-un punct M
aflat pe axa perpendiculară ridicată în centrul spirei, la distanța x de spiră.
OBS.: Datorită simetriei există două componente elementare de lungime dl, fiecare încărcate cu
densitatea de sarcină pz, care acționează în punctul M de calcul. Fiecare componentă va determina
un câmp electric elementar dE, respectiv dE'. Componentele tangențiale ale acestora se vor anula
reciproc, astfel încât această geometrie va determina un câmp electric doar cu o componentă

2na

dE cos a
o

Facultatea de Inginerie Electrică


2 na 2na

2Traxpz 1 axpi 1
4ttc0 r3 2e0 (a2 + x2)3/2

OBS.:
Dacă punctul de calcul M este plasatîn centrul spirei (x = 0) atunci câmpul electric produs de aceasta este nul.
Dacă geometria spirei este de tipul unui arc de cerc atunci formula generală se exprimă proporțional în raport
cu cercul complet (se înmulțește relația cu /i pentru un semicerc, cu % pentru un sfert de cerc).

Bazele Electrotehnicii I - Conf. dr. ing. Veronica PĂLTÂNEA, Facultatea de Inginerie Electrică
Aplicația 5
Un conductor electric filiform, având forma unei semi-spire circulare de rază a, încărcat cu
densitatea de linie p( > Ose află plasat în vid. Să se determine intensitatea câmpului electric E
într-un punct M aflat în centrul spirei.
OBS.: Datorită simetriei de tip semi-cerc există două componente elementare de lungime dl,
fiecare încărcate cu densitatea de sarcină pz, care acționează în punctul M de calcul. Fiecare
componentă va determina un câmp electric elementar dE, respectiv dE'. Componentele
tangențiale ale acestora se vor anula reciproc, astfel încât această geometrie va determina un câmp
electric doar cu o componentă normală, colineară cu axa Ox.

TI

E= -Bl- a sin a da = - -------


4rca2s0 J 4nae0 J
o o
Pi
—— (cos ir — cos 0) ==
4iraE0 2îrac0
Bazele Electrotehnicii I - Conf. dr. ing. Veronica PĂLTÂNEA, Facultatea de Inginerie Electrică
Exemplu
Un conductor electric filiform, având forma unui sfert de cerc de rază a, încărcat cu densitatea de
linie pz > 0 se află plasat în vid. Să se determine intensitatea câmpului electric E într-un punct M
aflat în centrul spirei.

Bazele Electrotehnicii I - Conf. dr. ing. Veronica PĂLTÂNEA, Facultatea de Inginerie Electrică
BAZELE
ELECTROTEHNICII I

Conf. Dr. Ing. Veronica PĂLTÂNEA

Bazele Electrotehnicii I - Conf. dr. ing. Veronica PĂLTÂNEA, Facultatea de Inginerie Electrică
Bazele Electrotehnicii I
Curs
Introducere. Mărimi electromagnetice
Legile teoriei macroscopice a electromagnetismului
Regimul electrostatic al câmpului electromagnetic
Regimul electrocinetic al câmpului electromagnetic
Regimul electrodinamic al câmpului electromagnetic
Seminar
Circuite electrice de curent continuu
Regimul electrostatic
Regimul electrodinamic

Bazele Electrotehnicii I - Conf. dr. ing. Veronica PĂLTÂNEA, Facultatea de Inginerie Electrică
Curs 3 Regimul electrostatic al
câmpului electromagnetic
3.5. Calculul câmpului electrostatic generat de corpuri cu simetrie
perfectă încărcate uniform cu sarcină electrică (Metoda Gauss)

^(S) - Q(d2)
(S)

Bazele Electrotehnicii I - Conf. dr. ing. Veronica PĂLTÂNEA, Facultatea de Inginerie Electrică
Tipuri de simetrii
Simetrie sferică Simetrie cilindrică (axială) Simetrie plană (plan-paralelă)

^sferei ~ 4ttf ^lat-cilindru ~ 2ttf/i A


^secțiune — A
~ 21

4ttf3
^cilindru ~~ ^paralelipiped
ferei ~ 5

Bazele Electrotehnicii I - Conf. dr. ing. Veronica PĂLTÂNEA, Facultatea de Inginerie Electrică
Aplicația 1
O sferă de rază a, încărcată cu o densitate de sarcină electrică pozitivă ps > 0 se află plasată în vid.
Să se determine inducția electrică D, intensitatea câmpului electric E și potențialul electric V în
interiorul si exteriorul sferei.

Sursa de câmp electric este o sferă încărcată cu ps > 0.


Liniile de câmp electric sunt radiale și pleacă de pe suprafața sferei.
Tipul de simetrie utilizat este simetria sferică. Pentru analiza la interior se va considera suprafața
închisă (5^) de forma unei sfere de rază r < a, iar pentru exterior se va considera suprafața închisă
(S2) de forma unei sfere de rază r > a.

Bazele Electrotehnicii I - Conf. dr. ing. Veronica PĂLTÂNEA, Facultatea de Inginerie Electrică 5
Cazul r > a

^2) = n(s2) cos(D, n(£z)) dz4 = D dA = DAnr2


(Z)

^S2) cL4 = ps4îra2


G^s/eră)

9 Psfl2
2=>D =i-£r-
^(S2) = Q(DE2) => ^4-nr2 = ps4na -y* z

r
Psa2
V-AP-,)
£0 r
00

Bazele Electrotehnicii I - Conf. dr. ing. Veronica PĂLTÂNEA, Facultatea de Inginerie Electrică
Cazul r < a

7
5.0354-407 : >5.3004-007
4.770002: 5.0H4-002
4 5054-407:4.7704-007
4 2404-407 : 4.5054-007
X9754-407 : 4.2404-007
3.7104-407: 3.9754-007
3.4454-407 : 3.7104-007
1180^407 : 3.4454-007
2 9154-407 : 3.1804-007
2 6504-40 7 : 2.9154-007
2-3854-407:2.6504-007
2120^407: 2.3854-007
1.8554-407 : 2.1204-007
1.5904-407 : 1.8554-007
1 325e407 : 15904-007
1.080*407 : 1.3254-007
7.9504-408 : 1.0604-007
5-3014-408 : 7.9504-008
2 6514-408 : 5.3014-008
<8.7434-012 : 2.6514-0»
Den*Y |0b G*«~2

OeMfy Mee v. vote

Potential, Volts*

Bazele Electrotehnicii I - Conf. dr. ing. Veronica PĂLTÂNEA, Facultatea de Inginerie Electrică 8
Aplicația 2
Un cilindru infinit lung de rază a, încărcat cu o densitate de sarcină electrică pozitivă ps > 0 se află
plasat în vid. Să se determine inducția electrică D, intensitatea câmpului electric E și potențialul
electric V în interiorul si exteriorul cilindrului.

OBS.:
Sursa de câmp electrostatic este un cilindru infinit lung încărcat cu ps > 0.

Cilindru infinit lung = se neglijează efectul suprafețelor plane, de


terminație, ale cilindrului finit.

Liniile de câmp electric sunt perpendiculare pe suprafața laterală,


orientate spre exterior.

Tipul de simetrie utilizat este simetria cilindrică. Pentru analiza la interior


se va considera suprafața închisă de forma unui cilindru de rază r < a
și înălțime h, iar pentru exterior se va considera suprafața închisă (S2) de
forma unui cilindru de rază r > a si înălțime h. I

Bazele Electrotehnicii I - Conf. dr. ing. Veronica PĂLTÂNEA, Facultatea de Inginerie Electrică
Cazul r > a S2 = U S2 U St; nsl 1 D; nS2 J- D

d/1 = D2nrh

I cL4 = ps2ixah
(■^cilindru)

pca
^(s2) = Q(dS2) => ^2-nrh = ps2nah ^D=~—

Bazele Electrotehnicii I - Conf. dr. ing. Veronica PĂLTÂNEA, Facultatea de Inginerie Electrică
Pentru calculul potențialului electric, deoarece structura este infinită, nu mai este posibilă alegerea
punctului de referință Po la infinit (V(co) 0).
Astfel se va considera că Po se găsește la o distanță r0 foarte departe de sursa de câmp electric, iar
valoarea potențialului în Po este V(r0).
p2 r
V2(P2) = V(P0)- f Ed(=>|/2(r)= |/(r0)- [—dr=»

Po ro
r
f 7dr = K(r0)- —(lnr-lnr0) = K(r0) ln^-
£0 J r £o £o ro
r0

Pentru simplificare se poate alege l/(r0) = 0.

Bazele Electrotehnicii I - Conf. dr. ing. Veronica PĂLTÂNEA, Facultatea de Inginerie Electrică
Cazul r < a = Si U S2 U St; nsl 1 D; nS2 -L D

^(21) = I cM = D2nrh
(Z)

dX = 0
(2i)

^(21) = <?cDSi)=>Z)2TTrh = 0

l'l(f’l) = Wo) - J Edl => (r) — V(r0 )


/>0
a
y (r) = yfr5) - — f-dr = K(r0) - — (In a - lnr0) = V(r0) -
£0 J r e0 s0 ro
r0

Bazele Electrotehnicii I - Conf. dr. ing. Veronica PĂLTÂNEA, Facultatea de Inginerie Electrică
Bazele Electrotehnicii I - Conf. dr. ing. Veronica PĂLTÂNEA, Facultatea de Inginerie Electrică
Aplicația 3
Un conductor filiform infinit lung, încărcat cu o densitate de sarcină electrică pozitivă pz > 0 se
află plasat în vid. Să se determine inducția electrică D, intensitatea câmpului electric E și
potențialul electric V determinate de conductor.

OBS.:
Sursa de câmp electrostatic este un fir infinit lung încărcat cu pz > 0.

Liniile de câmp electric sunt perpendiculare pe conductorul filiform,


orientate spre exterior.

Tipul de simetrie utilizat este simetria cilindrică. Pentru calculul mărimilor


electrice se va considera o suprafața închisă (S) de forma unui cilindru de
rază r si înălțime h.

Se poate aplica, de asemenea, formula integralei coulombiene


determinată anterior considerând = a2 — 0.

Bazele Electrotehnicii I - Conf. dr. ing. Veronica PĂLTÂNEA, Facultatea de Inginerie Electrică 14
S— U S2 U Si'f 1 D; nS2 -L D

I D n (sz) cL4 =
(St)

d/4 = D2nrh

{Lung ime_ci lindru)

D = Pi
D2nrh = p,h=^D =
HZ 2 nr s0 2n£0r

Bazele Electrotehnicii I - Conf. dr. ing. Veronica PĂLTÂNEA, Facultatea de Inginerie Electrică
Pentru calculul potențialului electric, deoarece structura este infinită, nu mai este posibilă alegerea
punctului de referință Po la infinit (V(co) 0).
Astfel se va considera că Po se găsește la o distanță r0 foarte departe de sursa de câmp electric, iar
valoarea potențialului în Po este V(r0).

Pentru simplificare se poate alege K(r0) = 0.

Bazele Electrotehnicii I - Conf. dr. ing. Veronica PĂLTÂNEA, Facultatea de Inginerie Electrică
Aplicația 4
Un plan infinit lung, încărcat cu o densitate de sarcină electrică pozitivă ps > 0 se află plasat în vid.
Să se determine inducția electrică D, intensitatea câmpului electric E și potențialul electric V
determinate de plan.

OBS.:
Sursa de câmp electrostatic este un plan infinit lung
încărcat cu ps > 0.

Liniile de câmp electric sunt perpendiculare pe plan,


orientate spre exterior.

Tipul de simetrie utilizat este simetria plan-paralelă.


Pentru calculul mărimilor electrice se va considera o
suprafața închisă (X) de forma unui paralelipiped cu
aria secțiunii transversalei și lungime 21.

Bazele Electrotehnicii I - Conf. dr. ing. Veronica PĂLTÂNEA, Facultatea de Inginerie Electrică
cL4 = ps4
(Arie_paralelipiped)

Ps
^(S) - Q(ds) =* 2D A - pSi4 =>D -
2

Bazele Electrotehnicii I - Conf. dr. ing. Veronica PĂLTÂNEA, Facultatea de Inginerie Electrică
Pentru calculul potențialului electric, deoarece structura este infinită, nu mai este posibilă alegerea
punctului de referință Po la infinit (V(co) 0).
Astfel se va considera că Po se găsește la o distanță x0 foarte departe de sursa de câmp electric, iar
valoarea potențialului în Po este K(x0).
X
l^dx

Xo

Pentru simplificare se poate alege x0 = 0 și V(x0) = 0-

Bazele Electrotehnicii I - Conf. dr. ing. Veronica PĂLTÂNEA, Facultatea de Inginerie Electrică
Aplicația 5
Se consideră fir infinit lung încărcat cu o densitate
de sarcină electrică pozitivă p; înconjurat de un
cilindru infinit lung de rază a, încărcat cu o
densitate de sarcină electrică pozitivă ps. Structura
se află plasată în vid.
Să se determine inducția electrică D, intensitatea
câmpului electric E și potențialul electric.

OBS.:
Se aplică metoda Gauss prin superpoziție.
Se consideră două cazuri:
r <a
r>a

Bazele Electrotehnicii I - Conf. dr. ing. Veronica PĂLTÂNEA, Facultatea de Inginerie Electrică
Pentru rezolvare se poate folosii un tabel unde se plasează variabila geometrică, de obicei raza
r și expresiile inducției electrice pentru fiecare geometrie simplă, care face parte din
superpoziție.
Pentru această problemă limita de interes este a, iar în superpoziție se vor utiliza expresiile
analitice obținute în cazul firului infinit lung, respectiv cilindrul infinit lung.

E_______ 0 a 00

0. Da O||
Firul infinit lung Pz Pl Pl
1
2nr 2 na 2nr
Cilindru de rază a 0 0 psa
Ps
r
Suma algebrică Pz Pl _L_ Pi ! Psa
1 2ira + ps
2nr 2w r

n - P*
1 2nr 1 27T£or

4 S + M = £" = “7 G + ps“)
Bazele Electrotehnicii I - Conf. dr. ing. Veronica PĂLTÂNEA, Facultatea de Inginerie Electrică 21
Aplicația 6
Se consideră structura din figură plasată in vid. Aceasta este formată
dintr-o sferă de rază b, cu permitivitatea electrică relativă sr,
încărcată uniform cu o sarcină electrică având densitatea de volum
pv. în interiorul sferei se realizează o cavitate vidată, neîncărcată
electric, de raza a(b > a).
Considerând metoda superpoziției să se calculeze expresiile
inducției electrice D și ale intensității câmpului electric E în cazurile:

A) r < a; B) a < r < b;C) r > b.

Calcul numeric: a = 2 cm,b = 5 cm,£r = 3, pv = 4000 pC/m3.

OBS.:
Se va considera metoda superpoziției, unde se vor aduna algebric o
structură sferică de rază b încărcată cu +pv și o structură sferică de
rază «încărcată cu —pv

Bazele Electrotehnicii I - Conf. dr. ing. Veronica PĂLTÂNEA, Facultatea de Inginerie Electrică
Simetria este de tip sferic, iar liniile de câmp electric au o distribuție radială

Raza 0 a b 00

-Ppr — ppa -Pv«3 -Pva3 -PvG3


Sferă de rază «încărcată cu —pv 0
3 3 3r2 3b2 3r2

Pp Ț Pvfl PvȚ Pvb Pv*3


Sferă de rază b încărcată cu +pv 0
3 3 3 3 372

Pvr Pva3 pvb pva3 Pv*3 pj,a3


Suma algebrică 0 0 0
3 3r2 3 3b2 3r2 3r2

k)r < a; Dl = 0; E^ = 0;
Pvr pp«3 1
B) a < r < b; Du = 3r2 '^n e0€r
3

pvb3 pX . r = 1 (Pyb3 pvcr


C) r > b; Dm =
3r2 ' 111 Eq \ 3r2 3r2

Bazele Electrotehnicii I - Conf. dr. ing. Veronica PĂLTÂNEA, Facultatea de Inginerie Electrică
3.6. Condensatorul electric.
3.6.1. Definiție.
/ Clasificare
Condensatorul electric este un sistem format din două corpuri conductoare omogene, numite
armături, între care există un mediu dielectric omogen sau neomogen, neîncărcat electric, fără
polarizație permanentă.
Când se aplică o tensiune între armături, acestea se încarcă cu sarcini egale și de semn contrar.
Mărimea ce caracterizează un condensator electric se numește capacitate electrică și se definește ca
raportul pozitiv dintre sarcina electrică care se încarcă pe o armătură și tensiunea electrică aplicată.
Valoarea indicată a capacității electrice a unui condensator este stabilită prin standard ca fiind
valoarea obținută în regim electrostatic.
Capacitatea electrică se notează cu C și are unitatea de măsură farad [F].

Bazele Electrotehnicii I - Conf. dr. ing. Veronica PĂLTÂNEA, Facultatea de Inginerie Electrică
Clasificarea condensatoarelor se face:
după forma armăturilor pot fi: plane, cilindrice, sferice, eliptice;
după posibilitatea varierii capacității pot fi: fixe, variabile, ajustabile;
C C C
HH /IZ
__
°“7
(a) (b) (c)
după natura dielectricului: cu aer, ulei, mase ceramice etc.
după destinație:
condensatoare utilizate în instalații energetice (compensarea factorului de putere, eliminarea
paraziților, aprinderea motoarelor cu explozie, divizoare capacitive de tensiune etc)
condensatoare utilizate în telecomunicații pentru realizarea acorduluiîn circuitelecu rezonanță,
pentru cuplajul etajelor succesive, scurtcircuitarea rezistențelorîn curent alternativ
după clasa de precizie
Clasa Toleranta
* fată
* de valoarea marcată
I
II 5%
III 10%
IV 20%

Bazele Electrotehnicii I - Conf. dr. ing. Veronica PĂLTÂNEA, Facultatea de Inginerie Electrică 25
3.6.2. Calculul capacității condensatorului plan
Condensatorul plan are distanța dintre armături mică în raport cu dimensiunea armăturilor astfel
încât poate fi neglijat efectul de capăt.

^(2) = J DdA = zEA


(S)
A = ab

=Q

= <7(Oj)=> = Q => E = Ș-

sA

Q Q zA
C =—
Uab Va - VB d
TĂd
Bazele Electrotehnicii I - Conf. dr. ing. Veronica PĂLTÂNEA, Facultatea de Inginerie Electrică
3.6.3. Calculul capacității condensatorului cilindric
Condensatorul cilindric are distanța dintre armături redusă în raport cu lungimea armăturilor astfel
încât poate fi neglijat efectul de capăt. I

f DdA = 2tkERK
(2)

- Q

^(S) = => ZmERh = Q E = —-—


2nsRh

2nzRh

Q Q Q 2mh 2mh
C Uab Va - Vb ...O-, ln^2
^cilindric ~ p~
2TT6/1111/?! m/?! ln^2
m/?1
Bazele Electrotehnicii I - Conf. dr. ing. Veronica PĂLTÂNEA, Facultatea de Inginerie Electrică 27
3.6.4. Calculul capacității condensatorului sferic
Este o configurație specială de condensator care nu are aplicație practică

V(l)
BAZELE
ELECTROTEHNICII I

Conf. Dr. Ing. Veronica PĂLTÂNEA

Bazele Electrotehnicii I - Conf. dr. ing. Veronica PĂLTÂNEA, Facultatea de Inginerie Electrică
Bazele Electrotehnicii I
Curs
Introducere. Mărimi electromagnetice
Legile teoriei macroscopice a electromagnetismului
Regimul electrostatic al câmpului electromagnetic
Regimul electrocinetic al câmpului electromagnetic
Regimul electrodinamic al câmpului electromagnetic
Seminar
Circuite electrice de curent continuu
Regimul electrostatic
Regimul electrodinamic

Bazele Electrotehnicii I - Conf. dr. ing. Veronica PĂLTÂNEA, Facultatea de Inginerie Electrică
Curs 3 Regimul electrostatic al
câmpului electromagnetic
3.6. Condensatorul electric
3.6.5. Teoremele Kirchhoff pentru rețele de condensatoare
Rețea de condensatoare - reprezintă un circuit format din condensatoare electrice și generatoare
ideale de tensiune continuă.
într-o rețea de condensatoare se definesc elemente topologice: nod, latură și buclă.
Alegerea semnului sarcinii electrice de pe o armătura a unui condensator se poate realiza în mod
aleator, dar, în consecință, va rezulta sensul căderii de tensiune la bornele acestuia, întotdeauna de la
armătura pozitivă către cea negativă. Reciproca este de asemenea validă.

+Q -Q
" Uc
Bazele Electrotehnicii I - Conf. dr. ing. Veronica PĂLTÂNEA, Facultatea de Inginerie Electrică
Teorema Kirchhoff I
Enunț: Suma algebrică a sarcinilor electrice de pe armăturile condensatoarelor care sunt
conectate într-un nod al rețelei este constantă

OBS.: Dacă se consideră condensatoarele initial


neîncărcate electric atunci suma algebrică este nulă.

(/)= ~<?1 + <?2 + <?3 - <?4 ~ <?S = °

Legea conservării sarcinii electrice


d(?(Ds)

- ĂT = constant
Î(S) = 0

Bazele Electrotehnicii I - Conf. dr. ing. Veronica PĂLTÂNEA, Facultatea de Inginerie Electrică
Teorema Kirchhoff II
Enunț: Suma algebrică a căderilor de tensiune de pe laturile care formează o buclă a unei
rețele
/ de condensatoare este nulă
K—1(A)
> ^ = 0

Forma extinsă pentru această relație este:


(A) = y(A) E](
2 c
Se demonstrează pe baza teoremei potențialului electrostatic
^04
Qi Q2 .Qz Qi Qs p p p
+ = Ex- E3- E4

Bazele Electrotehnicii I - Conf. dr. ing. Veronica PĂLTÂNEA, Facultatea de Inginerie Electrică
Aplicația 1
Să se determine sarcinile electrice și tensiunile condensatoarelor pentru circuitul din figură.
Se cunoaște capacitatea fiecărui condensator Q = C2 = C3 = C4 = C5 = C, iar tensiunile

Cl): - Qi + Q2 + Q3 - 0 (X)-.-C1U1 + C2U2 + C3U3 = 0 (1) : -U4 + U2 + U3 = 0


(2):-(?3 + (?4 + (?5 = 0 (2): —C3t/3 + C4t/4 + C5t/5 = 0 (2) : -U3 + U4 + U5 = 0
[I] : U1 + U2-U = E2 [I] : U4 + U2 - U = E2 [I] : l/i + l/2 - (/ = £
[II] : U3 + U4 - U2 = -E2 - E4 [II] : U3 + U4 - U2 = -E2 - E4 [II] : U3 + U4 - U2 = —2E
[III] : U5 - U4 = E4 [III] : U5-U4 = E4 [III] : U5-U4 = E

OBS.: (3): 4-^ - Q2 — Q4 - Q5 = 0 (nu face parte din sistemul de ecuații al problemei)
Bazele Electrotehnicii I - Conf. dr. ing. Veronica PĂLTÂNEA, Facultatea de Inginerie Electrică
[I]: t71 + t/2-t7 = E’^>t71=E, + (7-(/2=> (1): —E - U + U2 + U2 + U3 = 0 => U3 = E + U - 2U2
[III]: U5-U4 = E^U5 = E + U4

(2): -E - U + 2U2 + U4 + E + U4 = 0 => 2U2 + 2U4 = U\• (3)


[II]: E + U - 2U2 + U4 - U2 = —2E ^-3U2 + U4 = -3E - Z71 • (2)

U — GE U U 3U + 6E
8U4=3U-6E -2U = U-6E^U4= —-— U2=2~U*=2
4 4 8 8

5U + 2E 2U-4E U + 2E
8

Bazele Electrotehnicii I - Conf. dr. ing. Veronica PĂLTÂNEA, Facultatea de Inginerie Electrică
Aplicația 2
Să se determine sarcinile electrice și tensiunile condensatoarelor pentru circuitul din figură.
Se cunosc capacitățile fiecărui condensator Q = 2 [pF],C2 = 4 [|1F],C3 = 2 [|1F],C4 =
3 [țiF], C5 = 6 [țiF] și tensiunile electromotoare E± = E5 = 8 [V].

Topologia: n = 2, Z = 3,0 = /—n+l =

_(?2 - Qz ~ Q4 - 0 (1): —Q2 ~ Qz ~ Q4 - o


[I] : U± + U2 - U3 = Er [1]: 71 + ^“7Î = £i
[II] : —U3 + U4 + U5 = E5 L1 c2 l3
nn ^3 , Q4 , Qs c +
[H]:-—+ 7^ + 7^ =
Co Ca Ce

Datorita conexiunilor serie:


Bazele Electrotehnicii I - Conf. dr. ing. Veronica PĂLTÂNEA, Facultatea de Inginerie Electrică 8
1\ <?2 + <?4 [I] :?2|+?4|=S
U c3
[II] : ?21 + <?4 = 8
0.2 + @4
c3 +Qt(i+i)=E5
<?2 = 4 [pF] <?i = 4 [pF]
<?4 = 6 [pF] <?s = 6 [pF]
<?3 = -10 [pF]

U, = ^=2[V]
l4
Qs
u5 = c5 = 1 [V]

Bazele Electrotehnicii I - Conf. dr. ing. Veronica PĂLTÂNEA, Facultatea de Inginerie Electrică
3.6.6. Configurații echivalente de condensatoare
Conectarea serie a condensatoarelor

Configurațiile serie presupun o conservare a sarcini electrice pe fiecare armătură a condensatoarelor

Ql - Q2 -..... - Qn — Qe — Q
T.K. II: Ux + U2 +

Bazele Electrotehnicii I - Conf. dr. ing. Veronica PĂLTÂNEA, Facultatea de Inginerie Electrică
Conectarea paralel a condensatoarelor

Configurațiile paralel presupun o


conservare a tensiunii electrice la bornele
fiecărui condensator

U± = U2 =........ = Un = UAB = U

Qi + Q2 +..... + Qn - Qe Q^1 + c2u2 +..... + cnun - cp euAB => + c2 +

C± + C2 +........ + cn - Cp_e

Bazele Electrotehnicii I - Conf. dr. ing. Veronica PĂLTÂNEA, Facultatea de Inginerie Electrică
Transfigurarea stea-triunghi

C1C2 ^3 C3 -C1

+ C-, + Cj + C") + c3 C| + C 9 + C3

Bazele Electrotehnicii I - Conf. dr. ing. Veronica PĂLTÂNEA, Facultatea de Inginerie Electrică 12
Transfigurarea triunghi-stea
1

C12 C31 C23 C12 C31 C23


C*2 - C23 + Q2 + C3 -C31 + C23 +
C23 C12

Bazele Electrotehnicii I - Conf. dr. ing. Veronica PĂLTÂNEA, Facultatea de Inginerie Electrică 13
Bazele Electrotehnicii I - Conf. dr. ing. Veronica PÂLTÂNEA, Facultatea de Inginerie Electrică 14
3.6.7. Teorema divizorului capacitiv de tensiune

£ i qc2
Q
Q_ 1 CiC2
c2 ^2 Ql + ^2 Ci + c2
Bazele Electrotehnicii I - Conf. dr. ing. Veronica PĂLTÂNEA, Facultatea de Inginerie Electrică 15
3.6.8. Teorema divizorului capacitiv de sarcină electrică

<2i = tfiQ

Bazele Electrotehnicii I - Conf. dr. ing. Veronica PĂLTÂNEA, Facultatea de Inginerie Electrică 16
BAZELE
ELECTROTEHNICII I

Conf. Dr. Ing. Veronica PĂLTÂNEA

Bazele Electrotehnicii I - Conf. dr. ing. Veronica PĂLTÂNEA, Facultatea de Inginerie Electrică
Bazele Electrotehnicii I
Curs
Introducere. Mărimi electromagnetice
Legile teoriei macroscopice a electromagnetismului
Regimul electrostatic al câmpului electromagnetic
Regimul electrocinetic al câmpului electromagnetic
Regimul electrodinamic al câmpului electromagnetic
Seminar
Circuite electrice de curent continuu
Regimul electrostatic
Regimul electrodinamic

Bazele Electrotehnicii I - Conf. dr. ing. Veronica PĂLTÂNEA, Facultatea de Inginerie Electrică
Curs 3 Regimul electrostatic al
câmpului electromagnetic
3.7. Energii și forțe în câmp electrostatic
3.7.1. Energia câmpului electrostatic al unui sistem de corpuri conductoare
Principiului conservării energiei:
Energia elementară dWext primită din exterior de un sistem care suferă o transformare, este egală cu
suma dintre lucrul mecanic elementar 6L efectuat de sistem, căldura elementară 8Q dezvoltată,
creșterea elementară dWe a energiei sistemului și energia elementară dWt transformată în alte
forme:
= dL + 6Q + dWe + dWt

Pentru a putea definii o succesiune de stări electrostatice se consideră că transformarea elementară


se produce foarte lent și izoterm, fără dezvoltare sau transfer de căldură, și fără efectuarea de lucru
mecanic, astfel încât Energia câmpului electrostatic al unui sistem de corpuri conductoare este:
dWext = dWe
Bazele Electrotehnicii I - Conf. dr. ing. Veronica PĂLTÂNEA, Facultatea de Inginerie Electrică
Pentru ca sistemul de corpuri conductoare să primească energie din exterior este necesar să
se exercite o forță exterioară pentru a transporta sarcina electrică elementară de la infinit
până pe suprafața unui corp conductor.
Pentru un corp conductor k se consideră:
Punctul Pk de pe suprafața acestuia unde trebuie să ajungă sarcina electrică elementară
Sarcina electrică elementară care trebuie transportată dqk pe conductorul k
Forța exterioară Fkext aplicată
Lucrul mecanic elementar SLkext necesar pentru transportul de la infinit către conductorul k

Pentru întreg sistemul de n corpuri conductoare, lucrul mecanic elementar exterior necesar
pentru transportul sarcinii electrice elementare dqk pe fiecare conductor k în parte, este:
n n
$Lext ~~ * ^^kext “
k=l k=l
Bazele Electrotehnicii I - Conf. dr. ing. Veronica PĂLTÂNEA, Facultatea de Inginerie Electrică 4
în regim electrostatic se consideră că energia elementară primită de sistem din exterior este
doar sub formă de lucru mecanic:

Energia electrostatică totală încărcată în sistemul de n corpuri conductoare se calculează ca


integrala pe domeniul închis considerat:

Bazele Electrotehnicii I - Conf. dr. ing. Veronica PĂLTÂNEA, Facultatea de Inginerie Electrică
încărcarea electrostatică a conductoarelor electrice se realizează prin valori intermediare,
care definesc stări intermediare bine definite.
Aceste stări intermediare se pot caracteriza prin:
valori proporționale ale sarcinii electrice q'k = Xqk
valori proporționale ale potențialului electric V'k = XVk
unde Ă G (0,1), iar qk și Vk sunt valorile maximefinale.

Energia electrostatică totală a sistemului de n corpuri conductoare se scrie:

Ă=1 n / n \ 1 n n
we= I dWe= I ^V'kdq'k = /^VkqkjlMĂ = ^Vk<lk
A=0 &=1 \k = l / o ^=1 _ _ _ _ ^=1 __

Enunț:/
Energia câmpului electrostatic a unui sistem de conductoare încărcate este egală cu semisuma
produselor dintre potențialul și sarcina electrică a conductoarelor

Bazele Electrotehnicii I - Conf. dr. ing. Veronica PĂLTÂNEA, Facultatea de Inginerie Electrică 6
Pentru cazul particular al unui condensator electric format dintr-un sistem de două corpuri
conductoare încărcate cu sarcina electrică +q, respectiv -q, conectate fiecare la un potențial
electric V1, respectiv V2 are expresia energiei electrostatice:

Bazele Electrotehnicii I - Conf. dr. ing. Veronica PĂLTÂNEA, Facultatea de Inginerie Electrică
3.7.2. Densitatea de volum a energiei câmpului electrostatic
Caracterizarea locală a energiei electrostatice presupune definirea densității de energie
electrostatică we cu ajutorul mărimilor de stare ale câmpului electric și a elementelor geometrice
care definesc domeniul de integrare.

L. fluxului el.

1
we = -DE

Pentru medii liniare, omogene, izotrope și fără polarizație permanentă:

Bazele Electrotehnicii I - Conf. dr. ing. Veronica PĂLTÂNEA, Facultatea de Inginerie Electrică
3.7.3. Teoremele forțelor generalizate în câmp electrostatic
Se consideră un sistem de n conductoare încărcate cu sarcinile qk și având potențialele Vk, situate
într-un mediu dielectric, neîncărcat și infinit extins.
Poziția relativă a acestor conductoare poate fi descrisă prin intermediul a p coordonate generalizate,
x1,x2,---xp (mărimi scalare), reprezentate sub forma gradelor de libertate ale sistemului (lungime,
unghi, suprafață).
Asupra fiecărui conductor acționează o forța generalizată a cărei componentă Xj, poate să modifice
numai una dintre aceste coordonate generalizate Xj, lăsându-le pe celelalte nemodificate, ce poate
fi interpretată ca o forță newtoniană, cuplu sau tensiune superficială (presiune), în funcție de tipul
coordonatei generalizate.
Lucrul mecanic elementar asociat acestor forțe generalizate la modificări independente elementare
p
are expresia: 8L = Xjdxj
j=i
Energia elementară primită de cele n conductoare în regim electrostatic este:
n
ăWext = 2 Vkdqk =8L + dWe
k=l
Bazele Electrotehnicii I - Conf. dr. ing. Veronica PĂLTÂNEA, Facultatea de Inginerie Electrică 9
Sistemul de corpuri conductoare este izolat (qk = constant => dqk = 0)

=0 =0 8L + dlV, = 0 =» 8L =

Lucul mecanic este realizat prin scăderea energiei electrostatice interne a sistemului de
conductoare.
Dacă se exprimă energia electrostatică în funcție de sarcinile electrice de pe fiecare conductor și de
pozițiile coordonatelor generalizate ale corpurilor We(<q1,q2,--- qn>xi>x2>"' xp) atunci:
n
dWe 1 q2
-~~dqk
dqk
;=i ’ k=l ™
p V
8L = -(dWe)^ M x>=~
j=l j=l ' J ' qk=ct.
Forța generalizată Xj asociată coordonatei generalizate Xj este egală cu derivata parțială cu semn
schimbat a energiei electrostatice a sistemului (exprimată în funcție numai de sarcinile electrice și
de coordonatele generalizate), în raport cu coordonata generalizată Xj, la sarcini constante ale
conductoarelor.

Bazele Electrotehnicii I - Conf. dr. ing. Veronica PĂLTÂNEA, Facultatea de Inginerie Electrică 10
Sistemul de corpuri conductoare este conectat la surse de tensiune (are potențialul electric pe
fiecare conductor constat Vk = constant => dVk = 0)

n / n \ n n n
We = |VfcQfc => dWe = d (|ykqk \ = |Vkdqk +1 qkdvk = |Vkdqk
k=l \ k=l / k=l < k=l k=l

în relația de bilanț energetic avem:


n n n
dWext=^Vkdqk=SL+dWe=SL+^Vkdqk 8L=| £ Vkdqk = (dWe)Vk=ct,
k=l k=l k=l

Energia primită de sistem de la sursele exterioare se distribuie în mod egal pentru efectuarea de
lucru mecanic și pentru creșterea energiei electrostatice a sistemului.

Bazele Electrotehnicii I - Conf. dr. ing. Veronica PĂLTÂNEA, Facultatea de Inginerie Electrică 11
Dacă se exprimă energia electrostatică în funcție de potențialele electrice de pe fiecare conductor și
de pozițiile coordonatelor generalizate ale corpurilor W4(VÎ, yn>xi’x2>"' xp) atunci:

6L = (dH4)Kft=ct. = ix>
7=1 f=lV >'vk=ct.

Forța generalizată Xj asociată coordonatei generalizate Xj este egală cu derivata parțială a energiei
electrostatice a sistemului (exprimată în funcție numai de potențialele electrice și de coordonatele
generalizate), în raport cu coordonata generalizată Xj, la potențiale constante ale conductoarelor.
OBS.:
1. Pentru sistemele liniare, cele două expresii ale forței generalizate sunt echivalente.
2. Forțele electrostatice au valori mici, ceea ce face ca aplicațiile lor tehnice să se înscrie într-un
domeniu limitat, la construcția unor aparate de măsură și a unor traductoare.

Bazele Electrotehnicii I - Conf. dr. ing. Veronica PĂLTÂNEA, Facultatea de Inginerie Electrică
Moving
finger

Bazele Electrotehnicii I - Conf. dr. ing. Veronica PĂLTÂNEA, Facultatea de Inginerie Electrică
Aplicația 1
Se consideră un condensator plan, având dimensiunile din figură, care prezintă între armături
un dielectric cu permitivitate relativă er și grosime d. Armăturile condensatorului sunt
conectate la bornele unei surse de tensiune U.
Determinați expresia forței de tragere exercitate asupra armăturii inferioare necesare pentru
desprinderea acesteia.

Bazele Electrotehnicii I - Conf. dr. ing. Veronica PĂLTÂNEA, Facultatea de Inginerie Electrică 14
Sub acțiunea forței generalizate Fy are loc deplasarea armăturii inferioare cu distanța y
(coordonată generalizată).
Se observă că între armătura inferioară și blocul dielectric apare un volum cu aer care este
caracterizat prin permitivitatea relativă Er = 1. Această zonă determină fizic un alt tip de
condensator plan, care prezintă aceeași arie a armăturilor ca în configurația inițială, dar care
sunt conectate la alte valori de potențial electric (V superioară, respectiv V2 inferioară).
Această zonă determină un condensator plan notat C2.
După deplasarea armăturii inferioare, blocul dielectric paralelipipedic va determina o
configurație capacitivă cu armăturile conectate la potențialele V± și V. Această zonă
determină un condensator plan notat Q.

Bazele Electrotehnicii I - Conf. dr. ing. Veronica PĂLTÂNEA, Facultatea de Inginerie Electrică 15
Capacitatea condensatorului plan Cplan ~ d
zA zQzrab zozrab zoab
C2=~d d 1 CrC2 d y
zA zoab Q_s (y) C, +C->~ £0£rab , Epafr
c2= — d y
y y
Eozrab zoab zozrabzoab
d y dy £q zrab
ce_s(y) = zozrab zoab ~ zozraby + zoabd ~ d + zry
d y dy
U2 zozrab
wey-cejMu2= 2
U2 dce_s(y) U2 z0z2 ab
<o
2 dy 2 (d + zry)

Valoarea negativă a forței se explică prin faptul că la alimentarea condensatorului cu tensiunea U,


asupra armăturii inferioare acționează o forță verticală orientată în sus ce tinde să mențină armătura
lipită de blocul dielectric al condensatorului (de sens opusei celei calculate).
Bazele Electrotehnicii I - Conf. dr. ing. Veronica PĂLTÂNEA, Facultatea de Inginerie Electrică 16
Aplicația 2
Se consideră un condensator plan cu aria armăturilor A = a x b având distanța între armături
d, plasat în aer. Armăturile condensatorului sunt conectate la bornele unei surse de tensiune
(1 = 1^- V2. între armături pătrunde, pe o distanță oarecare x, un bloc prismatic format din
două straturi dielectrice de grosimi glt respectiv g2. Permitivitățile relative ale celor două
straturi sunt £rl și er2.
Determinați expresia forței exercitate asupra blocului dielectric.

Bazele Electrotehnicii I - Conf. dr. ing. Veronica PĂLTÂNEA, Facultatea de Inginerie Electrică 17
Bazele Electrotehnicii I - Conf. dr. ing. Veronica PĂLTÂNEA, Facultatea de Inginerie Electrică 18
9i

£0£r2^2 £0£r2

Bazele Electrotehnicii I - Conf. dr. ing. Veronica PĂLTÂNEA, Facultatea de Inginerie Electrică
BAZELE
ELECTROTEHNICII I

Conf. Dr. Ing. Veronica PĂLTÂNEA

Bazele Electrotehnicii I - Conf. dr. ing. Veronica PĂLTÂNEA, Facultatea de Inginerie Electrică
Bazele Electrotehnicii I
Curs
Introducere. Mărimi electromagnetice
Legile teoriei macroscopice a electromagnetismului
Regimul electrostatic al câmpului electromagnetic
Regimul electrodinamic al câmpului electromagnetic
Regimul electrocinetic al câmpului electromagnetic
Seminar
Circuite electrice de curent continuu
Regimul electrostatic
Regimul electrodinamic

Bazele Electrotehnicii I - Conf. dr. ing. Veronica PÂLTÂNEA, Facultatea de Inginerie Electrică
Curs 4 Regimul electrodinamic al
câmpului electromagnetic
4.1. Relații fundamentale în regim magnetic staționar
Regimul magnetic staționar: mărimile fizice nu variază în timp, dar există fenomene
termodinamice (există schimburi și transformări de energie).
Legea magnetizației temporare Afr = x?nw
Legea legăturii în câmp magnetic b = iionrH + noMp = b = pH
Legea conducției electrice £ + £( = p/

Bazele Electrotehnicii I - Conf. dr. ing. Veronica PÂLTÂNEA, Facultatea de Inginerie Electrică 3
Legea fluxului magnetic

‘t’(Z) = O => J BcL4 = O


GȘ)

Potențialul magnetic vector .4 (mărime matematică)

divB = 0=>B — rotA

/ de etalonare div rotA = 0 =>divA = 0


Condiția

Permite scrierea fluxului magnetic ca integrală pe un contur

Bazele Electrotehnicii I - Conf. dr. ing. Veronica PĂLTÂNEA, Facultatea de Inginerie Electrică 4
Legea circuitului magnetic (Teorema Ampere)

rotH = J

) = constant sau zero

Forme particulare ale curentului electric de conducție:


Se pot întâlnii diferite metode prin care un curent electric care parcurge un conductor electric
poate genera câmp magnetic. Astfel se definesc:
curent electric de conducție care parcurge un conductor electric filiform

curent electric de conducție care parcurge un conductor masiv

solenația sau curentul electric de conducție care parcurge un număr N de spire

Bazele Electrotehnicii I - Conf. dr. ing. Veronica PÂLTÂNEA, Facultatea de Inginerie Electrică 5
4.2. Tub de flux magnetic
Se numește tub de flux magnetic porțiunea de câmp delimitată de totalitatea liniilor de câmp
care trec prin toate punctele unei curbe închise.

Bazele Electrotehnicii I - Conf. dr. ing. Veronica PÂLiÂNEA, Facultatea de Inginerie Electrică 6
4.3. Teorema refracției liniilor de câmp magnetic
La trecerea printr-o suprafață de discontinuitate între două medii liniare și izotrope, fără
densitate de curent, cu permeabilități magnetice diferite respectiv p2, liniile de câmp
magnetic se refractă.
Calculând raportul dintre tangenta unghiului de incidență cq și a unghiului de refracție a2 se
obține:
Bit
tg«i _ fim _ Bit _ Ilicit _ Hi
tga2 B2ț B2ț P-2^2t H2
Bzn
L. leg. în câmp mag. => B — [iH
L. F. mag.=»fi1?I = B2n
L. circ. mag. =>Hlt = H2t

Relația arată că la trecerea dintr-un material cu permeabilitate magnetică foarte mare


(feromagnetic) într-unul cu permeabilitate magnetică mică (aer) (țq » p2), liniile de câmp
magnetic ies perpendicular pe suprafața corpului feromagnetic (a2 -»0 ).
Bazele Electrotehnicii I - Conf. dr. ing. Veronica PÂLTÂNEA, Facultatea de Inginerie Electrica
4.4. Calcului câmpului magnetic generat de corpuri parcurse de curent
electric de conducție (Relația Biot-Savart-Laplace BSL)

Relația BSL se aplică în cazul corpurilor parcurse de curent electric de conducție pentru care
variația sarcinilor electrice prin conductor este constantă în timp și nu se poate acumula sau anula.

Expresia generală a inducției magnetice a câmpului magnetic, generat de densitatea de curent /


care parcurge un corp masiv, într-un punct P aflat în spațiu și caracterizat prin vectorul de poziție r

Bazele Electrotehnicii I - Conf. dr. ing. Veronica PĂLTÂNEA, Facultatea de Inginerie Electrica 8
Pentru corpuri conductoare cu structură filiformă parcurse de curentul electric de conducție i plasate în vid,
relația
r BSL este:

OBS.: Direcția și sensul vectorului B sau H se determină cel mai ușor aplicând regula burghiului drept sau
regula mâinii drepte

Bazele Electrotehnicii I - Conf. dr. ing. Veronica PĂLTÂNEA, Facultatea de Inginerie Electrică 9
Aplicația 1
Un conductor filiform, rectiliniu, parcurs de curentul electric de conducție i se află plasat în vid. Să
se determine intensitatea câmpului magnetic H într-un punct M aflat la distanța x de conductor.
Capetele conductorului formează cu punctul de calcul unghiurile cq și a2.

H H = Hxi + Hyi + Hzk

dl X 7 = |dZ| |r| sin(dZ,r) u = rdl sin(îr - a) k = — rd/sin(a)k


Hx = Hi = 0 (k • i = 0; k 1 i)
H= Hy = Hj = 0(k-j = 0;klj)
Hz = Hk * 0 (k • k = 1)

Bazele Electrotehnicii I - Conf. dr. ing. Veronica PÂLTÂNEA, Facultatea de Inginerie Electrică 10
f rsina
I i șina
H7 = - — dl
4tt J r3 4-fț r2
(c)

l = x ctg a + d,=-(^Fla
x
r= —
sin a
z

I șina x
Wz = —
4tt / x \2 (șina)2
Vsina/
l
-— [cos(ir — a2) — cos aj = — -— [cos cti + cos a?l
4n% 4jtx
i
H = H7k = — -— cos a-, + cos a? k B = yt0Hzk = — [cos ai + cos a2]k
z 4nx 1 z

I
H = [cos at + cos a2] în funcție de orientarea vectorului H față de axele geometriei se alege ±
Bazele Electrotehnicii I - Conf. dr. ing. Veronica PĂLTÂNEA, Facultatea de Inginerie Electrică 11
Aplicația 2
Un conductor electric filiform, având forma unei spire circulare de rază a, parcurs de curentul
electric de conducție i se află plasat în vid. Să se determine intensitatea câmpului magnetic H într-
un punct M aflat pe axa perpendiculară ridicată în centrul spirei, la distanța x de spiră.
OBS.: Se utilizează din geometria 3D teorema
celor trei perpendiculare, astfel încât unghiul
dintre vectorul dZ și vectorul de poziție r este
de 90°
Datorită simetriei componentele pe axa Oyși Oz
sunt nule
dZ x r
^3

dl x r = | dZ| |r| sin(dZ, r) u = rdl sin u = rd/u


Hx = Hi Hy = Hj = 0

Hz = Hk = 0

Bazele Electrotehnicii I - Conf. dr. ing. Veronica PĂLTÂNEA, Facultatea de Inginerie Electrică 12
(di x r)i = rd/u • i = rd/|u||i| cos(u,i) = rd/ cos p = rd/ cos
a
sinet = —
r
= a2 + x2
u

4nr3 2r3 2(a2 + x2)3/2

ia2
H = Hxl = 2(a2 + x2)3/21

Mo B RoWx‘
H0ia2
2(a2 + x2)3/2*
* ’ r J - «_ _1 ,W —- __ - - bw

OBS.: Cazul în care punctul M este în centrul spirei (x = 0)


H = Hxi = ^-i B = RoC
„ 1
2a 2a
Pentru arce de cerc se înmulțește modulul cu proporția arcului de cerc din spira întreagă (semicerc = %)
Bazele Electrotehnicii I - Conf. dr. ing. Veronica PÂLTÂNEA, Facultatea de Inginerie Electrica 13
Aplicația 3
Un conductor electric filiform, având forma din figură, este parcurs de curentul electric de
conducție i fiind plasat în aer. Să se determine intensitatea câmpului magnetic H în punctul O,
considerând cunoscută dimensiunea a.

Bazele Electrotehnicii I - Conf. dr. ing. Veronica PÂLTÂNEA, Facultatea de Inginerie Electrică 14
»
I

i
4ttcz

H = -H1k= -~^—k
4ttcz

Bazele Electrotehnicii I - Conf. dr. ing. Veronica PĂLTÂNEA, Facultatea de Inginerie Electrică 15
Pentru segmentul (A, B)

I
[cos ax + cos a2]

x — a-, cq = 90°; a2 = 45°


«

i V2
Hi =----- [cos 90° + cos 45°1
4ira 4-rra 2

Bazele Electrotehnicii I - Conf. dr. ing. Veronica PĂLTÂNEA, Facultatea de Inginerie Electrică 16
Pentru segmentul (B, C) => H3

I

i V2
4-rra 2

Bazele Electrotehnicii I - Conf. dr. ing. Veronica PĂLTÂNEA, Facultatea de Inginerie Electrică 17
Bazele Electrotehnicii I - Conf. dr. ing. Veronica PĂLTÂNEA, Facultatea de Inginerie Electrică 18
Pentru segmentul (D,—co) =>H5

I
[cos cq + cos a2]
4nx

x = a; cq = 90°; a2 = 0°

I
Hi = -— [cos 90° + cos 0°l
4ira 4na
a

Bazele Electrotehnicii I - Conf. dr. ing. Veronica PÂLTÂNEA, Facultatea de Inginerie Electrica 19
Intensitatea câmpului magnetic în punctul O Ho
l

H0=Hxi + Hy] + Hzk =

Bazele Electrotehnicii I - Conf. dr. ing. Veronica PÂLTÂNEA, Facultatea de Inginerie Electrică 20
Pentru i = 1[A] și a = l[m]

https://www.monroecc.edu/faculty/paulseeburger/calcnsf/CalcPlot3D/

Bazele Electrotehnicii I - Conf. dr. ing. Veronica PĂLTÂNEA, Facultatea de Inginerie Electrică
4.5. Calculul câmpului magnetic generat de conductoare parcurse de curent
electric de conducție, care prezintă simetrie perfectă (Metoda Ampere)

umm^ = l(Sr) J JdA


C$r)
B = [iH

Metoda Ampere se aplică structurilor geometrice cu simetrie perfectă. Aceasta presupune existența unui
sistem de coordonate ales astfel încât datele problemei variază doar după una din direcțiile sistemului de
coordonate. în același timp se cunoaște forma liniilor de câmp magnetic.
Datele problemei:
• Configurația geometrică;
• Parametrii de material (permeabilitatea magnetică absolută, permeabilitatea magnetică relativă);
• Sursele de câmp magnetic (densitatea de suprafață/linie a curentului electric de conducție, curentul
electric de conducție)
Necunoscutele problemei:
• Expresiile matematice ale intensității câmpului magnetic H și a inducției magnetice B.
Bazele Electrotehnicii I - Conf. dr. ing. Veronica PÂLTÂNEA, Facultatea de Inginerie Electrică 22
Algoritmul metodei:
1. se stabilește tipul de simetrie și se desenează forma liniilor de câmp magnetic (curbe închise circulare,
concentrice, în jurul sursei de câmp magnetic), care se încadrează în tipul de simetrie;
2. se alege o curbă închisă T a cărei formă se încadrează în tipul de simetrie; se alege un punct de calcul P
aflat la intersecția liniilor de câmp magnetic cu T;
3. se calculează tensiunea magnetică prin curba închisă T:

4. se calculează curentul electric de conducție care trece prin interiorul suprafeței Sr:

densitate de curent (corpuri masive)

i(sr) - <
[ M‘= I Js<u=js I dl densitate superficială
(r) (r) (D
V l curent electric (conductoare filiforme)
OBS,: Domeniul de integrare efectiv este domeniul parcurs de curentul electric de conducție, de aceea poate să
nu fie identic cu domeniul Sr
Bazele Electrotehnicii 1 - Conf. dr. ing. Veronica PĂLTÂNEA, Facultatea de Inginerie Electrică 23
5. se aplică Teorema Ampere și se calculează expresia analitică a intensității câmpului magnetic H
u»ijn(r) = l(Sr)

6. se aplică Legea legăturii în câmp magnetic pentru corpuri liniare, omogene și izotrope, fără
magnetizație permanentă, și se calculează expresia analitică a inducției magnetice B:

B = [iH = poprH

Bazele Electrotehnicii I - Conf. dr. ing. Veronica PĂLTÂNEA, Facultatea de Inginerie Electrică 24
Aplicația 1
Un conductor cilindric, infinit lung, de rază a, având permeabilitatea pr, este parcurs de densitatea
de curent J fiind plasat în vid. Să se determine intensitatea câmpului magnetic H și inducția
magnetică fîîntr-un punct M aflat în interiorul, respectiv exteriorul, conductorului.

Simetrie cilindrică
Conductor masiv, parcurs pe întreaga arie a secțiunii transversale de J u0
Pentru caracterizare la interior se alege o curbă închisă de rază r < a
Pentru caracterizare la exterior se alege o curbă închisă r2 de rază r > a 1
I
Liniile de câmp magnetic sunt cercuri concentrice de rază r I
I
I
I
I
I

Bazele Electrotehnicii I - Conf. dr. ing. Veronica PÂLTÂNEA, Facultatea de Inginerie Electrică 25
Cazul r < a

ir
~ l(SP1) H ■ 2irr = J ■ vcr2 => H = —

/r
5 = pH = = Ho|.ir —

Bazele Electrotehnicii I - Conf. dr. ing. Veronica PĂLTÂNEA, Facultatea de Inginerie Electrică 26
Cazul r > a

/
dA = J ■ raz2
(^cilindru)

umm(r2) = ^r,)^ H • 2nr = J - ^a2 =>

B = [iH = ți0H = n0

Bazele Electrotehnicii I - Conf. dr. ing. Veronica PĂLTÂNEA, Facultatea de Inginerie Electrică 27
l$77«*O»: >4.055**000
3.70>HX0 : 3,$77e*O»
3.52>*O» : j.7CC<K<C
3J45<*O» : 3.»5fO»
3 IMeMWO : 3J4St*O»
2 991OGC0 : 316S«<CO
2-Sl3<*O» : 2-W«*O»
2.634<«O» : 2413««a»
2.45$e*O» : 2.636**0»
2.281e*O» : 2.4W**COO
2.10*4*O» : 2.2*lfO»
l.*27e*O»:2.10W*O»
l.Z49»*000: 1.927«*CCO
1.572**0» : 1.749**0»
1395**O» : X.572**O»
X.2S7e*O»: lJ9$**O»
!040*0» : l.217**O»
8 625^051 :1 W**OX
6.852e-O31 : 8.625e-001
<S.078e-001 :6-B52t-OX

length, mm

Bazele Electrotehnicii I - Conf. dr. ing. Veronica PÂLTANEA, facultatea de Inginerie Electrică 28
X.343e~000 : >X.404<*0(»
1 2834*000 : 1-3434*000
I 222e*000 : I 2S3e*<00
1.1624*000 : 1.2224*000
l.XOX««^OO : 1.1624*000
X.040e*000 : 1.10X4*000
9.7994-001 : 1040**000
9.19K-00X : 9.7994-001
8.587*001 : 9.193*001
7.9824-001 : 8.587*001
7.376*001 : 7.982*001
6.7704-001 : 7.3764-001
6.164*001 : 6.770*001
5.558*001 : 6.164*001
4.9524^01 : 5.556*001
4.347*001 : 4.952*001
3.741*001 : 4 >47*001
3.135*001 :3 741*001
2.5294-001 : 3.1354-001
<1.923*001 : 2.5294-001
D4o«yHcC: |B|. t*Uj

length, mm
11 r 1 1 ?
V V I1 I i I
:r li I I
r f
i i
1 V
J7 t rl t r h

|H|. Amp/m

length, mm

Bazele Electrotehnicii I - Conf. dr. ing. Veronica PÂLTANEA, Facultatea de Inginerie Electrica 29
Aplicația 2
Un conductor cilindric, infinit lung, de rază a, este parcurs la suprafață de densitatea de curent
superficial Js fiind plasat în vid. Să se determine intensitatea câmpului magnetic H și inducția
magnetică B într-un punct M aflat în interiorul, respectiv exteriorul, conductorului.

Simetrie cilindrică
Conductor gol, tubular, parcurs pe suprafața exterioară de către Js
Pentru caracterizare la interior se alege o curbă închisă de rază r < a
Pentru caracterizare la exterior se alege o curbă închisă r2 de rază r > a
Liniile de câmp magnetic sunt cercuri concentrice de rază r

Bazele Electrotehnicii I - Conf. dr. ing. Veronica PĂLTÂNEA, Facultatea de Inginerie Electrică 30
Cazul r < a

umma^ = i(sri) =>H-2nr = 0=>H = 0

Bazele Electrotehnicii I - Conf. dr. ing. Veronica PĂLTÂNEA, Facultatea de Inginerie Electrică
Cazul r > a

umm(r2} J dl = H ■ 2nr
(r2) (r2)

JsdA = I Jsdl= )s J dl = Js • 2 ir a

(sr2) (L cilindru) (^cilindru)

9
umm(r2) = l(Sr2) H • 2nr = Js • 2tuz => H = —
r

Bazele Electrotehnicii I - Conf. dr. ing. Veronica PÂLTÂNEA, Facultatea de Inginerie Electrică 32
Aplicația 3
Un conductor filiform, infinit lung, este parcurs de curentul electric de conducție i fiind plasat în
vid. Să se determine intensitatea câmpului magnetic H și inducția magnetică Fîntr-un punct M
aflat la distanta r de conductor.

Simetrie cilindrică
Conductor filiform, parcurs de curent electric de conducție i
Pentru caracterizare la exterior se alege o curbă închisă T de rază r
Liniile de câmp magnetic sunt cercuri concentrice de rază r

dl = H • 2nr

l(sr) = 1
9
l
= <(sr)=> * 2nr = i =>// = ^ £? = pH = p0H = p0 —

Bazele Electrotehnicii I - Conf. dr. ing. Veronica PĂLTÂNEA, Facultatea de Inginerie Electrică 33
BAZELE
ELECTROTEHNICII I

Conf. Dr. Ing. Veronica PĂLTÂNEA

Bazele Electrotehnicii I - Conf. dr. ing. Veronica PĂLTÂNEA, Facultatea de Inginerie Electrică
Bazele Electrotehnicii I
Curs
Introducere. Mărimi electromagnetice
Legile teoriei macroscopice a electromagnetismului
Regimul electrostatic al câmpului electromagnetic
Regimul electrodinamic al câmpului electromagnetic
Regimul electrocinetic al câmpului electromagnetic
Seminar
Circuite electrice de curent continuu
Regimul electrostatic
Regimul electrodinamic

Bazele Electrotehnicii I - Conf. dr. ing. Veronica PĂLTÂNEA, Facultatea de Inginerie Electrică
Curs 4 Regimul electrodinamic al
câmpului electromagnetic
4.5. Calculul câmpului magnetic generat de conductoare parcurse de curent
electric de conducție, care prezintă simetrie perfectă (Metoda Ampere)
/ Hdl= / B = \1H
(r) (sr)
Metoda Ampere se aplică structurilor geometrice cu simetrie perfectă. Aceasta presupune existența unui
sistem de coordonate ales astfel încât datele problemei variază doar după una din direcțiile sistemului de
coordonate. în același timp se cunoaște forma liniilor de câmp magnetic.
Datele problemei:
• Configurația geometrică;
• Parametrii de material (permeabilitatea magnetică absolută, permeabilitatea magnetică relativă);
• Sursele de câmp magnetic (densitatea de suprafață/linie a curentului electric de conducție, curentul
electric de conducție)
Necunoscutele problemei:
• Expresiile matematice ale intensității câmpului magnetic H și ale inducției magnetice B.
Bazele Electrotehnicii I - Conf. dr. ing. Veronica PĂLTÂNEA, Facultatea de Inginerie Electrică 3
Algoritmul metodei:
1. se stabilește tipul de simetrie și se desenează forma liniilor de câmp magnetic (curbe închise circulare,
concentrice, în jurul sursei de câmp magnetic), care se încadrează în tipul de simetrie;
2. se alege o curbă închisă T a cărei formă se încadrează în tipul de simetrie; se alege un punct de calcul P
aflat la intersecția liniilor de câmp magnetic cu T;
3. se calculează tensiunea magnetică prin curba închisă T:

4. se calculează curentul electric de conducție care trece prin interiorul suprafeței Sr:

densitate de curent (corpuri masive)

densitate superficială

curent electric (conductoare filiforme)


OBS.: Domeniul de integrare efectiv este domeniul parcurs de curentul electric de conducție, de aceea poate să
nu fie identic cu domeniul Sr
Bazele Electrotehnicii I - Conf. dr. ing. Veronica PĂLTÂNEA, Facultatea de Inginerie Electrică
5. se aplică Teorema Ampere și se calculează expresia analitică a intensității câmpului magnetic H

umm(r) ~ l(Sr)

6. se aplică Legea legăturii în câmp magnetic pentru corpuri liniare, omogene și izotrope, fără
magnetizație permanentă, și se calculează expresia analitică a inducției magnetice B:

B = \iH = p0 [irH

Bazele Electrotehnicii I - Conf. dr. ing. Veronica PÂLTÂNEA, Facultatea de Inginerie Electrică
Aplicația 4
Se consideră structura din figură plasată în vid. Aceasta este formată dintr-o coroană cilindrică infinit extinsă
realizată dintr-un material feromagnetic conductor electric, de rază exterioară a, respectiv rază interioară b,
parcursă uniform de o densitate a curentului electric de conducție J, care înconjoară un fir filiform infinit lung,
plasat în axa de simetrie a cilindrului, parcurs de un curent electric de conducție i.
Considerând metoda superpoziției să se calculeze intensitatea câmpului magnetic H și inducția magnetică B în
cazurile:

A) r < b; B) b < r < a; C) r > a.

Calcul numeric: a = 1 mm, b = 0.7 mm,/ = 5 MA/m2, i = 1 kA.


Mo
Simetrie cilindrică
Coroană cilindrică masivă, feromagnetică, parcursă pe întreaga arie a secțiunii transversale de J
Pentru caracterizarea A) r < b se alege o curbă închisă 1/ de rază r < b
Pentru caracterizarea B) b < r < a se alege o curbă închisă r2 de rază b < r < a
Pentru caracterizarea C) r > a se alege o curbă închisă f3 de rază r > a
Liniile de câmp magnetic sunt cercuri concentrice de rază r
Bazele Electrotehnicii I - Conf. dr. ing. Veronica PĂLTÂNEA, Facultatea de Inginerie Electrică
Se aplică metoda superpoziției, care va avea treji structuri geometrice de analizat

Geom. III

b l
____ L_____

-r.
- _i_ -x
[ (Ti)

l Mo
I_____ Geom. I Geom. II

Bazele Electrotehnicii I - Conf. dr. ing. Veronica PĂLTÂNEA, Facultatea de Inginerie Electrică
Razar 0 b a oo

Geom. 1 1 = 2nr H' = 2nr H' = 2^

Geom. II Jr Ja2
o ttnjnt = y flnjnt = y Hn-ext ~ 2r

Geom. III Jr Jb2 Jb2


o #III_int — ~ y ^Ill.ext — 2r ^Ill.ext — 2r

Suma algebrică 1 HA=-^— i Jr Jb2 i Ja2 Jb2
' A 2irr 2rrr 2 2r 2irr 2r 2r

A) r < b => HA =

Q)b<r<a=>HB =

C) r > a

Bazele Electrotehnicii I - Conf. dr. ing. Veronica PĂLTÂNEA, Facultatea de Inginerie Electrică
2.0tW«0M 1 >2.1«14»000
|JtW«000 : 2,«W*000
1»4*-OX> : l.MWOOO
ITU* *000 : :.»M««O0O
UttHMtlUNHOBO
l.WWOOO : inmoo
I.MI*«O00: 1COWOOO
L41MKW : 1,$1U«000
UHtHNlMlStHN
1724»«X>: l.)lft«OQO
172to«000 : 1724* *000
t.O32fttO : 1.121* *000
4)64*-C01 : 1 OWeK-»
f.408+401 : 9•-364+00I
7.4+5+401 : 81.406+001
6.492+401 : 7.449+001
3.335+401 : 64We001
4.32W-001 : 3-333+001
1620+401 : 4.5W+001
<?W>W1 : 1620+401
D»<WfrHot: Ill.TnU

Bazele Electrotehnicii I - Conf. dr. ing. Veronica PĂLTÂNEA, Facultatea de Inginerie Electrică 9
4.6. Bobina electrică
4.6.1. Definiție. Clasificări.
/

Bobina electrică este un dispozitiv realizat dintr-un conductor filiform izolat electric, dispus astfel încât să
formeze un număr de spire înseriate care să înlănțuie același suport.
Când se aplică un curent electric prin conductor bobina generează câmp magnetic.
Mărimea ce caracterizează o bobina este inductivitatea electrică și se definește ca raportul dintre fluxul
magnetic produs de bobină și curentul electric care parcurge conductorul acesteia.
Inductivitatea electrică se notează cu L și are unitatea de măsură henry [H].

Suportul pentru realizarea unei bobine poate fi:


din material nemagnetic (plastic, aer). în acest caz câmpul magnetic se distribuie liber în mediul din jurul
bobinei. Apare fenomenul de dispersie magnetică, care are valori mari
din material magnetic. în acest caz câmpul magnetic este menținut în jurul bobinei sau este ghidat spre
zonele de interes. Fenomenul de dispersie magnetică este foarte redus.

Bazele Electrotehnicii I - Conf. dr. ing. Veronica PÂLTÂNEA, Facultatea de Inginerie Electrică
Reprezentare simbolică pentru cazul bobinei ideale (rezistența electrică a conductorului se
consideră nulă) - dacă se consideră ca bobina are N spire atunci:

Fluxul magnetic total: <t> = N<p

X /
Fluxul magnetic produs de o singură spiră a bobinei se numește//ux
\ /
fascicular <p
<p = BA
în cazul unei singure bobine se definește inductivitatea electrică:
Ncp
L =-=—
Clasificare: 1 1
• După funcționare: înfășurările mașinilor si transformatoarelor
electrice, bobine de șoc (filtrare), bobine de aprindere (la motoare
cu explozie), bobine de stingere (Petersen, utilizate în energetică),
bobine de acord
• După frecvența de lucru: de frecvență înaltă, de frecvență joasă și de
/ \
frecvență industrială (50 Hz, 60 Hz)
/ \ • După natura suportului: cu și fără miez de fier.
Bazele Electrotehnicii I - Conf. dr. ing. Veronica PĂLTÂNEA, Facultatea de Inginerie Electrică
Cazul bobinelor cuplate magnetic- se consideră un sistem format de n bobine electrice astfel încât:
■ se consideră bobina j caracterizată de: curentul ij, numărul de spire Nj
■ se consideră bobina k caracterizată de: curentul ik, numărul de spire Nk
Pentru calculul parametrilor de cuplaj se consideră o singură bobină parcursă de curent. Astfel avem ik = 0.

Definiții:
r
• tpkj - flux magnetic fascicular de cuplaj (mutual) produs de
curentul ij printr-o spiră a bobinei k
• (pd j - flux magnetic fascicular de dispersie (scăpări) produs
de curentul ij, dar ale cărui linii de câmp nu intersectează nici
o altă bobină din configurație
• <p;;- - flux magnetic fascicular total produs de curentul ij, care
parcurge o spiră a bobinei j
Pentru o singură bobină j din sistemul format de n bobine
electrice, fluxul fascicular total se scrie:
n

k=l
k*j
Bazele Electrotehnicii I - Conf. dr. ing. Veronica PĂLTÂNEA, Facultatea de Inginerie Electrică
Definiții:
• Lkj - inductivitatea de cuplaj (mutuală) produsă de curentul ij prin spirele bobinei k (ik = 0)
! _ _ ^k^kj n
lj ~ i-
L*i~ i- Lj
• Ldj - inductivitatea de dispersie (scăpări)

_ *d,j _ NjVd.j
Ld,j- lji- ~ i-
Lj >0
• Ljj - inductivitatea proprie a bobinei j

=="JVJJ > 0
JJ 17• 7 •
7

Atunci când ambele bobine sunt parcurse de curent se obține:

Conform teoremei Neumann Ljk = Lkj astfel încât se definește coeficientul de cuplaj magnetic:

e [0,1]

Bazele Electrotehnicii I - Conf. dr. ing. Veronica PĂLTÂNEA, Facultatea de Inginerie Electrică
n
Lkjij Nj
k = A Nk ij

Definiții:
• LUk - inductivitatea utilă a bobinei j (se consideră deoarece este o mărime pozitivă prin definiție)

Observație: Ldj « LUkJ

Bazele Electrotehnicii I - Conf. dr. ing. Veronica PĂLTÂNEA, Facultatea de Inginerie Electrică
Semnul (sensul) cuplajului magnetic (al inductivității de cuplaj) se pune în evidență cu ajutorul bornelor
polarizate (steluțelor) plasate la una din extremitățile fiecărei bobinei și a unui arc de cerc.
Cuplajul magnetic dintre două bobine este pozitiv (diferențial) atunci când sensul curenților prin bobine
parcurg în același mod bornele polarizate și negativ (aditiv) atunci când sensul curenților prin bobine parcurg
în mod diferit bornele polarizate.
Pentru bobina j cuplată magnetic cu bobina k pot exista două cazuri asociate la patru configurații:

cuplaj magnetic pozitiv cuplaj magnetic negativ

Ik Lk Ik

Bazele Electrotehnicii I - Conf. dr. ing. Veronica PĂLTÂNEA, Facultatea de Inginerie Electrică
4.6.2. Calculul inductivitătii unui tor.
Torul este o bobină circulară care nu prezintă fenomenul de dispersie magnetică. Liniile de câmp magnetic se
distribuie uniform în interiorul suportului spirelor

Bazele Electrotehnicii I - Conf. dr. ing. Veronica PĂLTANEA, Facultatea de Inginerie Electrică 16
Pentru cazul r < R

umm(ri)

0(SF1) = 0

2irrH = 0=>H = 0=>B = O

în interiorul torului câmpul magnetic este nul

Bazele Electrotehnicii I - Conf. dr. ing. Veronica PÂLTÂNEA, Facultatea de Inginerie Electrică 17
Pentru cazul r > R + 2a

Bazele Electrotehnicii I - Conf. dr. ing. Veronica PĂLTÂNEA, Facultatea de Inginerie Electrică 18
Pentru cazul R < r < R + 2a

umm(r) i(sr) 0(sr)

2îirH = Ni^H =-—


2 Tir

Bazele Electrotehnicii I - Conf. dr. ing. Veronica PĂLTÂNEA, Facultatea de Inginerie Electrică 19
Calculul inductivității (R < r < R + 2a)
p/Vi
B=^=h

Fluxul fascicular produs de o singură spiră este:

Pentru geometria dată: ) = na

Fluxul total produs de tor este: cp = /v<p =

0 p/V2i p/V
Inductivitatea torului: L = — = ------ Arc = -—Arc
i 2nri(5torJ 2nr ( tor)
p/V 2 p/V2 _ pa 2N2 pa2/V2 2 pa2/V2
în cazul geometriei date: L = „ ~—na2 —
2irr tor 2nr 2r 2 /? + /? + 2a 2(R + a)
R + R + 2a
unde am considerat raza medie a torului: r =
2
Bazele Electrotehnicii I - Conf. dr. ing. Veronica PĂLTÂNEA, Facultatea de Inginerie Electrică 20
4.6.3. Calculul inductivitătii unui solenoid.
Solenoidul este o bobină dreaptă cu lungimea mult mai mare decât raza, care permite obținerea unui câmp
magnetic ale cărui linii sunt paralele și distribuite uniform în secțiunea transversală a bobinei.

Valori cunoscute: l,r,(l » r),A,ți,i,N


Deoarece l » r se poate neglija fenomenul de dispersie
magnetică astfel încât se consideră că, în exteriorul AU
®®®®®®...... ®®®...... ®®® ®®® X
structurii, câmpul magnetic este nul. UI

---------- P--------- '----- --------- # >----- * V


Pentru simplificare se consideră că nu există un efect de
capăt semnificativ.
2)®$®®® ............ ®®S>.............. ®®®

Se aplică teorema Ampere pe o curbă închisă, care trece


prin interiorul și exteriorul structurii cilindrice, pentru a
calcula intensitatea câmpului magnetic H .-i «L... UiUla . L c-a «afli < 1 iitti MiLj

Bazele Electrotehnicii I - Conf. dr. ing. Veronica PĂLTÂNEA, Facultatea de Inginerie Electrică
Pentru identificarea sensului liniilor de câmp magnetic se utilizează regula mâinii drepte.
Curba închisă f se consideră formată din două curbe deschise Q1 9și C2.

N,i
®®®®&®...... ®®®......

_y

(CO 0(Sr) = Ni
2) ® ® ® ® <■ Ni Ni
— l(Sr) — ®G$r) NI — Ni=> H — — B = ,~
<

JJ./VÎ
Fluxul fascicular produs de o singură spiră este: <p =
/
(Șsolenoid)
/M/1 =
/
(Ssolenoid)
/?cL4 ^^(sS0ien0id) A

Pentru geometria dată: A = w2


u.N2i
Fluxul total produs de solenoid este: <t> = Ncp =------- A

0 [xN2i p/V2 p4 _
Inductivitatea solenoidului: L= — A = —-A=^-N2
i li

Bazele Electrotehnicii I - Conf. dr. ing. Veronica PĂLTÂNEA, Facultatea de Inginerie Electrică 22
4.6.4. Calculul inductivității mutuale dintre configurații geometrice de conductoare filiforme.
Se consideră că sursele de câmp magnetic sunt conductoare filiforme parcurse de curent electric realizate sub
forma unui fir infinit lung și/sau a unui cadru cu o geometrie dată.
Valori cunoscute: dimensiunile geometrice,
Pentru calculul inductivității mutuale L21 se consideră i2 = 0,
dar se identifică direcția și sensul inducției magnetice B2
determinate de acesta în interiorul suprafeței mărginite de
cadrul filiform, utilizând regula mâinii drepte.
Câmpul magnetic produs de firul infinit lung se va calcula la o
distanță oarecare r de acesta. Cu regula mâinii drepte se
determină sensul și direcția vectorului inducție magnetică, iar
modulul acestuia este:
PoH

Se calculează fluxul magnetic de cuplaj (produs de inducția


magnetică prin suprafața mărginită de cadrul f2)

Bazele Electrotehnicii I - Conf. dr. ing. Veronica PĂLTÂNEA, Facultatea de Inginerie Electrică
Se calculează inductivitatea de cuplaj L21

Aplicația 1
Se consideră structura din figură plasată în vid. Aceasta este formată dintr-un conductor filiform infinit lung
parcurs de curentul de conducție și un cadru dreptunghiular parcurs de curentul electric i2 aflat la distanța
x de acesta. Să se calculeze inductivitatea de cuplaj dintre cele două conductoare.

Bi = o
2irr
[ 1*11 (Sr2))d/12 -
*^21 — n(Sr2) C0S
(Sr2)

x+b x+b
x+b
= 51cos(0°)d/l2 =
Roli j Poaii
-— aar = —— f ldr=^ln
j zur ZTl J r l -TT X
($r2) X X

, _ ^21 poa x+b


L21 — In-------
Bazele Electrotehnicii I - Conf. dr. ing. Veronica PĂLTÂNEA, Facultatea de Inginerie Electrică
BAZELE
ELECTROTEHNICII I

Conf. Dr. Ing. Veronica PĂLTÂNEA

Bazele Electrotehnicii I - Conf. dr. ing. Veronica PĂLTÂNEA, Facultatea de Inginerie Electrică
Bazele Electrotehnicii I
Curs
Introducere. Mărimi electromagnetice
Legile teoriei macroscopice a electromagnetismului
Regimul electrostatic al câmpului electromagnetic
Regimul electrodinamic al câmpului electromagnetic
Regimul electrocinetic al câmpului electromagnetic
Seminar
Circuite electrice de curent continuu
Regimul electrostatic
Regimul electrodinamic

Bazele Electrotehnicii I - Conf. dr. ing. Veronica PĂLTÂNEA, Facultatea de Inginerie Electrică
Curs 4 Regimul electrodinamic al
câmpului electromagnetic
4.7. Circuite magnetice
4.7.1. Definiții.
/ Clasificări
Materiale magnetice
După modul în care se comportă la introducerea într-un câmp magnetic exterior există materiale:
■ liniare: diamagnetice (%m < 0, pr ~ 0.999, sunt respinse de câmpul magnetic)
paramagnetice (xm > 0, pr ~ 1.009, sunt atrase de câmpul magnetic)
■ neliniare: feromagnetice (xm > 0, pr » 1)
ferimagnetice
antiferomagnetice
Materialele magnetice din clasa fero- și feri- magnetice prezintă fenomenul de histerezis magnetic și sunt
caracterizate de ciclul de histerezis.
Materiale magnetice moi fero și feri magnetice sunt utilizate pentru ghidarea și concentrarea câmpului
magnetic (|ir = 500 h- 50000).
Materiale magnetice dure fero și feri magnetice sunt utilizate pentru producerea câmpului magnetic
((pr = 1.5 4- 10) magneți permanenți)
Bazele Electrotehnicii I - Conf. dr. ing. Veronica PĂLTÂNEA, Facultatea de Inginerie Electrică 3
4.7.1. Definiții. Clasificări
f

Sursele de câmp magnetic: bobine parcurse de curent electric și/sau magneți permanenți.
în practică, pentru realizarea unor valori ridicate ale inducției și fluxului magnetic, elementele de generare a
câmpului magnetic (bobinele parcurse de curenți și magneții permanenți) sunt dispuse sau inserate între
piese realizate din materiale magnetice moi.
Circuitul magnetic reprezintă o asociere continuă dintre piese realizate din materiale magnetice moi, surse de
câmp magnetic și eventuale întreruperi transversale tehnologice realizate din material nemagnetic.
Zonele de material nemagnetic se numesc tehnic întrefieruri.
Primary and Secondary
'.Vndnțs
-I /

----------- _---------------1 Magnetic ------------------------------

Calculul circuitelor magnetice se face în regim staționar sau cvasistaționar al câmpului magnetic și se
bazează pe câteva ipoteze simplificatoare:
Se neglijează dispersia magnetică - închiderea liniilor de flux se realizează în totalitate prin circuitul magnetic
în orice secțiune transversală a circuitului magnetic fluxul magnetic se consideră a fi uniform distribuit.

Bazele Electrotehnicii I - Conf. dr. ing. Veronica PĂLTÂNEA, Facultatea de Inginerie Electrică 4
4.7.2. Relația Ohm pentru circuite magnetice. Reluctanța magnetică
Se consideră o porțiune de circuit magnetic format dintr-un segment realizat din material magnetic moale cu
permeabilitatea pr, liniar, omogen și izotrop, fără întrefieruri, parcurs de un flux magnetic fascicular <p, având
lungimea 1 și aria secțiunii transversalei.

B B <p
B = yiH^H = - =------ <p = BA => B =
R kom- A

B 1 <p
Um(C) ------ dl =
PoPr

Relația Ohm pentru circuite magnetice: uni(C) ~

Se definește reluctanța magnetică (rezistență magnetică): Rm = I ------- -dl


J PoPM
(c)
Pentru cazul unui mediu liniar, omogen ș izotrop, având aria secțiunii transversale constantă pe întreaga
lungime se obține: l I
Rm PoM M
Bazele Electrotehnicii I - Conf. dr. ing. Veronica PÂLTÂNEA, Facultatea de Inginerie Electrică 5
Pentru cazul în care segmentul de circuit magnetic prezintă și o bobină electrică având solenația 0 = Ni,
unde N este numărul de spire al bobinei, iar i este curentul electric de conducție care parcurge o spiră a
bobinei, relația Ohm se poate scrie sub forma generală, considerând Teorema Ampere.

Asemănătorcircuitelor electrice se poate realiza o reprezentare simbolică a unui tronson de circuit magnetic

unde pentru sensul solenației s-a utilizat regula mâinii drepte.

Bazele Electrotehnicii I - Conf. dr. ing. Veronica PĂLTÂNEA, Facultatea de Inginerie Electrică
4.7.3. Teoremele Kirchhoff pentru circuite magnetice
A. Elemente de topologie a circuitelor magnetice
Nodul - intersecția a minim trei segmente de circuit magnetic parcurse de către un flux magnetic fiecare

Latura - porțiunea de circuit magnetic dintre două noduri

Bucla - perimetru închis format din laturi de circuit magnetic

Bazele Electrotehnicii I - Conf. dr. ing. Veronica PĂLTÂNEA, Facultatea de Inginerie Electrică
4.7.3. Teoremele Kirchhoff pentru circuite magnetice
B. Teorema Kirchhoff I pentru circuite magnetice
Suma algebrică afluxurilor magnetice care parcurg laturile unui circuit magnetic ce aparțin unui nod este nulă.
OBS: teorema K I se aplică de obicei considerând fluxurile fasciculare prin laturile nodului

Se consideră un nod de circuit (/) căruia îi aparțin un număr de laturi de circuit magnetic parcurse de fluxuri
fasciculare, care au sensuri bine definite.
Se aplică Legea Fluxului Magnetic pe o suprafață închisă X definită în jurul nodului (/), care intersectează
fiecare latură de circuit magnetic printr-o suprafață deschisă (Sr/f), unde k este indicele laturi respective.

(/): -<p! + <p2 -<p3 -<p4 + <p5

Bazele Electrotehnicii I - Conf. dr. ing. Veronica PĂLTÂNEA, Facultatea de Inginerie Electrică
4.7.3. Teoremele Kirchhoff pentru circuite magnetice
C. Teorema Kirchhoff II pentru circuite magnetice
Suma algebrică a solenațiilor care înlănțuie laturile unei bucle de circuit magnetic este egală cu suma
algebrică a tensiunilor magnetice din lungul laturilor care aparțin buclei respective (produsul dintre reluctanța
magnetică a laturii și fluxul magnetic fascicular prin latura respectivă).
OBS: teorema K II se aplică în regim de funcționare staționar și cvasistaționar, pentru fiecare latură de circuit
magnetic considerând dispersia magnetică nulă

Se consideră o buclă de circuit [b] căruia îi aparțin un număr de laturi de circuit magnetic având reluctanțele
magnetice Rmk, parcurse de fluxuri fasciculare q>k, care au sensuri bine definite. Pe laturile respective se pot
regăsii bobine ale căror solenații sunt Qk.

Se aplică Teorema Ampere pe o curbă închisă rd definită prin laturile care aparțin buclei [b]. Fiecare latură

din buclă se caracterizează printr-o curbă deschisă Ck astfel încât rd = Ck, unde k este indicele laturi

respective.

Bazele Electrotehnicii I - Conf. dr. ing. Veronica PĂLTÂNEA, Facultatea de Inginerie Electrică
mm(rb) ~ 0(Srb)

y~“i (A)
mm(rd) = A

7—7

q>2

Ww3 Z
(A) y—1 (A)
/ Rmk<Pk ~ /
P4 'Vke[b] ^k,lke[b]

[^J: ^m2^2 V<P3 + Rm4<?4 ^m5^5 ~~ ©i + 0

Schema echivalentă de circuit este:

Bazele Electrotehnicii I - Conf. dr. ing. Veronica PĂLTÂNEA, Facultatea de Inginerie Electrică
Analogie între circuite electrice și circuite magnetice

Circuite electrice Circuite magnetice


Densitatea de curent J Inducția magnetică B
Intensitatea câmpului electric E Intensitatea câmpului H
magnetic

Intensitatea curentului electric / Fluxul magnetic fascicular <P
Tensiunea electrică u Tensiunea magnetică
Rezistenta electrică R Reluctanța magnetică
Conductanta electrică G Permeanța magnetică A
Tensiunea electromotoare e Solenatia 0

Bazele Electrotehnicii I - Conf. dr. ing. Veronica PĂLTÂNEA, Facultatea de Inginerie Electrică 11
Aplicația 1
Se consideră un circuit magnetic cu o geometrie conform figurii. Acesta este realizat dintr-un material
feromagnetic având permeabilitatea relativă pr. Pe două tronsoane verticale sunt realizate bobine electrice
caracterizate prin parametrii Nx,ix, respectiv N2,Î2- Cea de a treia coloana verticală prezintă un întrefier
având lățimea 6.
/

Să se calculeze:
Fluxurile magnetice fasciculare prin laturile circuitului;
Intensitatea câmpului magnetic și inducția magnetică prin laturile circuitului;
Inductivitatea proprie și de cuplaj a bobinei 1;
Inductivitatea proprie și de cuplaj a bobinei 2;
Fluxurile magnetice fasciculare de dispersie și inductivitatea de dispersie.

(vedere frontală)
Bazele Electrotehnicii I - Conf. dr. ing. Veronica PĂLTÂNEA, Facultatea de Inginerie Electrică
OBS: Pentru calculul circuitelor magnetice se va considera câmpul magnetic uniform distribuit pe întreaga
secțiune transversală constantă a unui tronson de lungime dată. Astfel, lungimile diferitelor porțiuni de
circuit magnetic se vor calcula ca lungimi medii ale geometriei respective.

(vedere laterală)

Circuitul magnetic are 2 noduri (B, E) și 3 laturi (BAFE, BCDE, BE). Se identifică lungimile medii ale
tronsoanelor cu aceeași arie. Dacă pe lungimile respective apar variații ale proprietăților magnetice ale
materialului suport atunci se va considera lungimea medie a materialului respectiv.
<AB-<BC-<DE-'EF-y. /CD1 =/CD2 =;

^fa — ^be — Z6 = 5;
Bazele Electrotehnicii I - Conf. dr. ing. Veronica PĂLTÂNEA, Facultatea de Inginerie Electrică 13
Schema de circuit asociată este:

7?ab Rbc
Aria laturilor de circuit magnetic este constantă A = a - g
Reluctanțele magnetice ale diferitelor porțiuni de circuit sunt:
©1 ©2 k *2
Rab = Rbc = Rde = Rbf = A 2^ag
R
11 *1
/?FA — ^BE — a ~
(p1, <P2 RqD2 PoM
I1-____
--- 11 I
R-FE Red lA-8
? *i — 8
Rcni — RcD2 — 7 —
HoM 2[ioiirag
8 5
RS =------- 7 =-------
PoM Vo“g

Solenațiile sunt: 0-l = 02 = N2i2


Bazele Electrotehnicii I - Conf. dr. ing. Veronica PĂLTÂNEA, Facultatea de Inginerie Electrică 14
—i—J—>-L '
Sistemul de ecuații
/ Kirchhoff:
©z ■Kcdi
^)®2

— '<7
fe
(B):-<pi - <p2+<P3= 0 KP3
] M[

&
•^FA As

1

[I]: (Â’aB + ^FA + ^FE^^Pl - ^BE(P2 = ©1 — ©2 <P1' k <p2. -^CD2 (B)

[II]: ^BE(P2 + (^BC + ^CDl + Zd2 + ^Ed)^ + ^8<P3 = ©2 Ăfe (E) 7?ed ©1 ©2
±93

(B):-<p! -<p2+(p3=0 91- 92-

(E)
[I]:(2— -----+ ——) h
<Pi------------- <p2 = N^- N2 i2 [I]:7?mi<Pi - ^m2<p2 = N1i1 - N2i2
\ 2g0|irag HoPrW Ihlh-ag

[H]:«m2<P2 + ^m3<P3 = ^2

Bazele Electrotehnicii I - Conf. dr. ing. Veronica PÂLTÂNEA, Facultatea de Inginerie Electrică 15
(B): <p3 = <Pi +<p2

[I] :/?TO1(pl - /?m2(pz = N1i1 ~ N2Î2 [I]: ~ Rm2Rm3^>2 = Rm3Wl ~ ^2)

[II] :/?m3(pl + (/?TO2 + ^7M3)<p2 — ^2^2 [n]: —2?77i1Z?77i3 <P1 — ^m1(^?7i2 + ^7773)^2 = ~Rm1^2^2

( ^m2^m3 ^m1^m2 ^m1^m3)(^)2 “ ^m3^lh ^2?2) ^mx^2?2

^2 _p p _ p p
2 v m 2 m3 m 1 m2 Rm± ^m3

[I]: — ^m2<P2 = ^171 ~ ^2^2 W: ^m1(^m2 + ^m3)(Pl — ^m2(^ m2 + Rm3)<P2 ~ (^77i2 + ^77i3)(^lil “ N2i2)

[H]:fi,„3<Pi + (fi,„2 + R,„3)<P2 = N2l2 [II]:R,„2R,„3(P1 + R„12(R,„2 + R,„3)<p2 = R„t2N2i2

(^m1^m2 + ^m1^m3 + Rm2#7713)<P1 - (^777 2 + ^7773)(^1z1 ~ ^2^2) + Rm2 ^2^2

(Rm2 + ^77i3)(^lh ” N2l2) +


<P1 = ^?n1^??i2 + ^m1^m3 + ^m2^m3

Bazele Electrotehnicii I - Conf. dr. ing. Veronica PĂLTÂNEA, Facultatea de Inginerie Electrică 16
)(W1i1-W2i2) + Rm2W2i2 - N2i2)^ RmiN2i2
^m2^m3 + + ^m1^m3

(^m2 + ^m3 ) (^1 h N2i2) + ^m2^2^2 ^m3(^lh ^2^2) + ^mx^2^2


(P3 =
^m^m2 mim3

^m2^lh + ^^2^2 r™rui


<P3 = R R +R R +R R [Wbl
7771
rYm 1 ^m
rY7772 ' /A7771 /A7773 ' rY7772rY7773

RmN2i2 ~ N2i2)
(p? = — [Wb]J
T2 Rm2^m3 _1_D D
' rv7H1rv77l2
ID O
' 2Y?7l11V77l3
L

«m2Niii + - N2i2)
[Wb]
^m^m2 + ^m^m3 + ^m2^m3

Bazele Electrotehnicii I - Conf. dr. ing. Veronica PĂLTÂNEA, Facultatea de Inginerie Electrică 17
Intensitatea câmpului magnetic și inducția magnetică prin laturile circuitului;

Prin fiecare latură a circuitului magnetic valoarea inducției magnetice este:


<P1 <P1
= [T]
A “ ag
^2
B2 = [T]
A ag
<P3 ._ <P3
[T]
A

Pentru calculul intensității câmpului magnetic trebuie să se considere proprietățile magnetice ale
materialului. Astfel, se va nota HFe pentru tronsoanele din material magnetic, respectiv H8 pentru cazul
zonelor cu întrefier.
fîi
IP1 [A/m] b3 (Do
HFel = — Hpe^ — — [A/m]
Mo Mr MoMz-a#
„ _ B2 _ <P2<P2 fA
Hpe2 —------ — [A/m] H8 = —[A/m]
Mo Mr MoMr«0 Mo • 1 Mo «5
Bazele Electrotehnicii I - Conf. dr. ing. Veronica PĂLTÂNEA, Facultatea de Inginerie Electrică 18
Inductivitatea proprie și de cuplaj a bobinei 1;

OBS: Pentru calculul inductivităților de cuplaj trebuie să fie activă o singură bobină din circuitul magnetic

Calculul inductivității proprii și de cuplaj a bobinei 1 presupune i2 = 0. Se pasivizează solenația 02

Schema de circuit obținută este:

Se poate calcula reluctanța echivalentă la bornele solenației 0X


Rm2Rm3

nm2 ' nm3

Fluxul fascicular propriu este:


01 Afiii
<P11 _

am e P I m2 m3 + ^m3) + ^m2^m3
nm1 D 1 D
nm2 ' n
m3

Bazele Electrotehnicii I - Conf. dr. ing. Veronica PĂLTÂNEA, Facultatea de Inginerie Electrică
Fluxul fascicular de cuplaj este (se aplică regula divizorului de flux magnetic asemănătoare cu regula

divizorului de curent):

Fluxul fascicular de dispersie se calculează utilizând aceeași regulă a divizorului de flux magnetic:

_ ^m2 _
<p31 - 911 p—777 “
nm 2 ' nm 3
_ ^l?l(^m2 + ^m3)

+ ^m3) + ^m2^m3

Bazele Electrotehnicii I - Conf. dr. ing. Veronica PĂLTÂNEA, Facultatea de Inginerie Electrică
Inductivitatea proprie:

_ Ni<Pn N1 (,Rm2 + Rm3)


Lu -
h 1 2 =o + ^m3) + ^m2^m3

Inductivitatea de cuplaj prin bobina 2 determinată de bobina 1:

_ ^2^21 Rm3NlN2 ______


l21 ~
h i2=0 + R m2Rm3

Inductivitatea de dispersie a bobinei 1:

^1<P31 R,n2N}
^d,l —
l'l i2=0 Ri 1(^77i2 + ^J7i3) + Rm2Rm3

Bazele Electrotehnicii I - Conf. dr. ing. Veronica PĂLTÂNEA, Facultatea de Inginerie Electrică
Inductivitatea proprie și de cuplaj a bobinei 2;

Calculul inductivității proprii și de cuplaj a bobinei 2 presupune = 0. Se pasivizează solenația 0t

Schema de circuit obținută este:

Se poate calcula reluctanța echivalentă la bornele solenației 02


(B) _ RmiRm3
Rme ~ Rm2 + R^ + R^
')©2
932
Fluxul fascicular propriu este:
] w
■Kml ^m2 Rial

912 922.
1 ^2 ^2 (^7?^ + ^7R3)

(E)
+ ^tr3) + ^77i1^77i3

Bazele Electrotehnicii I - Conf. dr. ing. Veronica PĂLTÂNEA, Facultatea de Inginerie Electrică
Fluxul fascicular de cuplaj este:

N2i2(Rm1 + ^m3) ^m3 R m3^2^2


<P12 — <P22
^tr33 )
Rm2(.Rm1 +1 “ + ^7n1^7R3 (<R77i1 + ^tr3) R77i2(.R77i1 "F ^7r3) "F ^7rx^7r3

Fluxul fascicular de dispersie se calculează utilizând aceeași regulă a divizorului de flux magnetic:

<P32 <P22 n Ț"B “


txm3
N2i2^Rm1 + ^îr3) Rm1
^7R2(^7R1 "F R 7R3) + ^7R1^7R3 (<R77l1 + ^7R3)

R 771^2^2
^tr2wtrx "F ^m3) + ^7rx^7r3

Bazele Electrotehnicii I - Conf. dr. ing. Veronica PĂLTÂNEA, Facultatea de Inginerie Electrică 23
Inductivitatea proprie:

_ ^2^22 + #m3)
L22 —
z2 ii=0 + ^m3) + Rm1Rm3

Inductivitatea de cuplaj prin bobina 1 determinată de bobina 2:

Rm^N,
l12 —
*2 ^m2^m1 + ^m3) +
ix=o i ™3

Inductivitatea de dispersie a bobinei 2:

_ N2<P32 RmiN?
Ld,2 —
z2 ji=0 + ^m3) + ^m^m3

Bazele Electrotehnicii I - Conf. dr. ing. Veronica PĂLTÂNEA, Facultatea de Inginerie Electrică
OBS: Cele două expresii obținute pentru inductivitățile de cuplaj sunt egale

_ iV2<p21 ^m3^1^2 ^m3^1^2


L21 —
ii i2=0 ^m1(^?n2 + i^m3) + ^m2^m3 ^m1^m2 + ^mr^m3 + ^m2^m3

Rm3NrN2 Rm3NiN2
l12 —
z2 i1=0 ^m2(^m1 ^™3) + ^m1^m2 + ^m1^m3 + ^m2^m3

Bazele Electrotehnicii I - Conf. dr. ing. Veronica PĂLTÂNEA, Facultatea de Inginerie Electrică 25
BAZELE
ELECTROTEHNICII I

Conf. Dr. Ing. Veronica PĂLTÂNEA

Bazele Electrotehnicii I - Conf. dr. ing. Veronica PĂLTÂNEA, Facultatea de Inginerie Electrică
Bazele Electrotehnicii I
Curs
Introducere. Mărimi electromagnetice
Legile teoriei macroscopice a electromagnetismului
Regimul electrostatic al câmpului electromagnetic
Regimul electrodinamic al câmpului electromagnetic
Regimul electrocinetic al câmpului electromagnetic
Seminar
Circuite electrice de curent continuu
Regimul electrostatic
Regimul electrodinamic

Bazele Electrotehnicii I - Conf. dr. ing. Veronica PĂLTÂNEA, Facultatea de Inginerie Electrică
Curs 4 Regimul electrodinamic al
câmpului electromagnetic
4.8. Energii și forțe în câmp magnetic staționar
4.8.1. Energia câmpului magnetic generat de un sistem de conductoare filiforme
Principiului conservării energiei:
Energia elementară dWext primită din exterior de un sistem care suferă o transformare, este egală cu suma
dintre lucrul mecanic elementar 8L efectuat de sistem, căldura elementară SQ dezvoltată, creșterea
elementară dV7m a energiei sistemului și energia elementară transformată în alte forme:

dWext =SL + SQ + dWm + dWZt

Se consideră un sistem de n circuite electrice filiforme,


fiecare spiră de ordin k (k = 1 h- n) fiind caracterizată
printr-o rezistență electrică intrinsecă Rk, având aplicată
la borne o tensiune electrică uk, parcursă de un curent
electric de conducție ik ce va genera un flux magnetic

Bazele Electrotehnicii I - Conf. dr. ing. Veronica PĂLTÂNEA, Facultatea de Inginerie Electrică 3
Energia câmpului magnetic al întregului sistem de conductoare filiforme este:
n n
^ukikdt = ft/c^dt + 6L + dWm
k=l

Legea conducției electrice aplicată unui circuit k, considerând Ufk tensiunea electrică în lungul conductorului
filiform și ek tensiunea electromotoare a circuitului, se scrie:
ufk + ek = Rkik==*ufk = Rkik

Curba închisă (r) definită de conturul conductorului filiform și linia tensiunii electrice aplicate la borne,
permite scrierea Legii inducției electromagnetice. Expresia acesteia este:
d<J> k
er = uf - uk = —-r-~ >uk = Rkik +
JK dt ufk=Rkik dt
Relația de mai sus, considerând tot sistemul de conductoare și înmulțind fiecare termen cu ikdt, devine:
n n n n n n
* uk ~ ^k^k + /?k^dt + ^\dd>k
u^i^dt =
dt
k=i k=i k=l k=l k=l k=i

Bazele Electrotehnicii I - Conf. dr. ing. Veronica PĂLTÂNEA, Facultatea de Inginerie Electrică 4
Comparând expresiile inițiale și finale ale densității de energie primită de la exterior:
n n
I
z
Lr — 1
Rfcifcdt + 8L + dWm n
SL + dlV„ = Y ik
k=l

Considerând sistemul de conductoare filiforme imobil, astfel încât nu există transformări prin care să se
efectueze lucru mecanic (8L = 0), iar curenții și fluxurile magnetice acționează prin valori intermediare,
care definesc stări intermediare bine definite, putem scrie:
valori proporționale ale curentului electric de conducție i'k = Aifc
valori proporționale ale fluxului magnetic = A3>k
unde A G (0,1), iar ik și sunt valorile maximefinale.
Energia magnetică produsă de sistemul de conductoare parcurse de curent electric este:

Bazele Electrotehnicii I - Conf. dr. ing. Veronica PĂLTÂNEA, Facultatea de Inginerie Electrică 5
Enunț: Energia câmpului magnetic a unui sistem de conductoare filiforme parcurse de curent electric este
egală cu semisuma produselordintre curentul electric și fluxul magnetic asociat fiecărui conductorfiliform.
" n

k=l
Pentru cazul unei bobine se va considera n = 1, (a nu se confunda cu numărul de spire al bobinei), astfel
încât energia magnetică produsă de o bobină este:

1 Li 2 (J)2
HC = - î0> — —
?n 2 2 2L
Pentru cazul a două bobine cuplate magnetic, utilizând relația de definiție în funcție de inductivitate și
curent, se obține:

Energia magnetică de interacțiune între bobine

/
Energia magnetică proprie a bobinei 1 Energia magnetică proprie a bobinei 2
Bazele Electrotehnicii I - Conf. dr. ing. Veronica PĂLTÂNEA, Facultatea de Inginerie Electrică 6
4.8.2. Densitatea de volum a energiei câmpului magnetic
Caracterizarea locală a energiei magnetice presupune definirea densității de volum a energiei magnetice wm
cu ajutorul mărimilor de stare ale câmpului magnetic și a elementelor geometrice care definesc domeniul de
integrare.
Utilizând expresiile mărimilor de stare ale câmpului magnetic în cazul unui solenoid, cu N spire, lungime l și
aria secțiunii
f transversalei:
= Mp = NBA 1 1HI 1 1z
<p Ni
^ = 2 i4,= 2V NBA = - HBIA =
=~
b a H =-
i=T
HI 2

^n = |^ = J Hdlț BdA = I ^HBdldA J ^HBdV = J wmdV


r S D% D%
Def. Fluxului magnetic

magnetic
/ țBHdV w„,=^H
Dt

Pentru medii liniare, omogene, izotrope și fără magnetizație permanentă: M'- = ^|//2 = B
2m2
Bazele Electrotehnicii I - Conf. dr. ing. Veronica PĂLTÂNEA, Facultatea de Inginerie Electrică 7
OBS:
Comparație între densitatea de volum a energiei electrice și densitatea de volum a energiei magnetice
Densitatea de volum a energiei electrice (valori uzuale) E = 10 [kV/cm], e0 = 4 * 1q9 [F/mJ

1 IO8
----------------- 10 • 10^ 4,42 [J/m3]
4tt • 9 • 109

Densitatea de volum a energie magnetice (valori uzuale) B = 1 [T], p0 = 4it • 10 7|H/m]

B2 _ 1 1 IO7
400000 [J/m3]
2p0 2 4tt • IO"7 8tt

Se observă că densitatea de volum a energiei magnetice este de aproximativ 90.000 de ori mai mare decât a
celei electrice, ceea ce justifică importanța aplicațiilor tehnice și domeniile largi de utilizare a dispozitivelor
magnetice.

Bazele Electrotehnicii I - Conf. dr. ing. Veronica PĂLTÂNEA, Facultatea de Inginerie Electrică 8
4.8.3. Teoremele forțelor generalizate în câmp magnetic
Se consideră un sistem de n conductoare filiforme parcurse de curent electric de conducție ik ce va
genera un flux magnetic
Poziția relativă a acestor conductoare poate fi descrisă prin intermediul a p coordonate generalizate,
x1,x2>'"^p (mărimi scalare), reprezentate sub forma gradelor de libertate ale sistemului (lungime,
unghi, suprafață).
Asupra fiecărui conductor acționează o forța generalizată a cărei componentă Xj, poate să modifice
numai una dintre aceste coordonate generalizate Xj, lăsându-le pe celelalte nemodificate, ce poate
fi interpretată ca o forță newtoniană, cuplu sau tensiune superficială (presiune), în funcție de tipul
coordonatei generalizate.
Lucrul mecanic elementar asociat acestor forțe generalizate la modificări independente elementare
p
are expresia: 6L = Xjdxj
j=i
Același lucru mecanic elementar, din expresia bilanțului energetic în câmp magnetic, este definit:
n
8L = Y‘k d*» - d%,
k=l
Bazele Electrotehnicii I - Conf. dr. ing. Veronica PÂLTÂNEA, Facultatea de Inginerie Electrică 9
Sistemul de conductoare filiforme generează fluxuri magnetice care sunt menținute constant
(d>fe = constant => = 0)

Lucul mecanic este realizat prin scăderea energiei magnetice a sistemului de conductoare.
Dacă se exprimă energia magnetică în funcție de fluxurile magnetice, generate de fiecare conductor,
și de pozițiile coordonatelor generalizate ale acestora VKn((I>1,<t,2> "• atunci:

Forța generalizată Xj asociată coordonatei generalizate Xj este egală cu derivata parțială cu semn
schimbat a energiei magnetice a sistemului (exprimată în funcție numai de fluxurile magnetice și de
coordonatele generalizate), în raport cu coordonata generalizată Xj, la fluxuri magnetice constante.

Bazele Electrotehnicii I - Conf. dr. ing. Veronica PĂLTÂNEA, Facultatea de Inginerie Electrică 10
Conductoarele filiforme sunt parcurse de curenți electrici constanți
(ik = constant =$ dik = 0)

n n n

—IZ k=l
<&kik =>dWm = d
k=l
2
k=l
^kdik = -
k=l

în relația de bilanț energetic avem:


n n n n
«A = £ ik dd>t - d%, = £ ik d*fc -| £ ifcd*k = |Ș\d<t>t 6L = Ct
k=l k=l k=\ k=l

Energia primită de sistem de la sursele exterioare se distribuie în mod egal pentru efectuarea de
lucru mecanic și pentru creșterea energiei magnetice a sistemului.

Bazele Electrotehnicii I - Conf. dr. ing. Veronica PĂLTÂNEA, Facultatea de Inginerie Electrică 11
Dacă se exprimă energia magnetică în funcție de curentul electric, care parcurge fiecare conductor,
și de pozițiile coordonatelor generalizate ale conductoarelor , in>xi>x2>'"xp) atunci:

6L = (dWTO)ifc=ct

Forța generalizată Xj asociată coordonatei generalizate Xj este egală cu derivata parțială a energiei
magnetice a sistemului (exprimată în funcție numai de curentul electric și de coordonatele
generalizate), în raport cu coordonata generalizată Xj, la curent electric constant prin conductoare.
OBS.:
Pentru sistemele liniare, cele două expresii ale forței generalizate sunt echivalente.

Bazele Electrotehnicii I - Conf. dr. ing. Veronica PĂLTÂNEA, Facultatea de Inginerie Electrică 12
Aplicația 1
Se consideră un circuit magnetic cu o geometrie conform figurii. Acesta este realizat dintr-un material
feromagnetic având permeabilitatea relativă infinită pr -> oo, fiind alcătuit din două componente. O structură
de forma literei E (armătură fixă) și o componentă de forma literei I (armătura mobilă). Intre cele două
armături se găsește un întrefier cu lățimea 8. Pe coloana verticală centrală este realizată o bobină electrică
caracterizată prin parametrii
Să se calculeze:
Fluxurile magnetice fasciculare prin laturile circuitului;
Intensitatea câmpului magnetic și inducția magnetică prin laturile circuitului;
Inductivitatea proprie a bobinei 1;
Fluxurile magnetice fasciculare de dispersie și inductivitatea de dispersie;
Energia magnetică dezvoltată în întrefier; a ■A Ai
Forța magnetică dezvoltată în întrefier. armătura fixă
00

------- :--------- armatură


<----------------------------
/
(vedere frontală) (vedere laterală)

Bazele Electrotehnicii I - Conf. dr. ing. Veronica PĂLTÂNEA, Facultatea de Inginerie Electrică 13
OBS: Deoarece permeabilitatea magnetică relativă a materialului din care este realizată partea magnetică a
circuitului este infinită atunci reluctanțele magnetice asociate respectivelor tronsoane de circuit magnetic vor
fi nule.
Schema de circuit magnetic echivalentă va conține doar reluctanțele magnetice ale tronsoanelor din întrefier.

Circuitul magnetic are 2 noduri și 3 laturi.


Reluctanțele magnetice din zonele cu întrefier sunt:
8 8
^81 —--------- 7 —---------
PoM \ioag

6 6
^82 ~ a ~
PoM Voag
8 8
/?83 —--------- - —---------
PoM noag
Solenația este: = Niii

Bazele Electrotehnicii I - Conf. dr. ing. Veronica PĂLTÂNEA, Facultatea de Inginerie Electrică 14
Fluxurile magnetice fasciculare prin laturile circuitului
Sistemul de ecuații Kirchhoff:

(2): (pi - <p2-<p3 = 0

[I]:/?5<P1 +/?6(p2 = ©!

[n]:7?6<Pi +/?6<p3 = ©i

Sistemul de ecuații cu metoda curentilor de buclă:


/ f

[<PblJ: (^8 + ^s)<Pbl + ^8<Pb2 = ®1


[<Pb2J: (^8 + ^s)(Pb2 + KS<Pbl = ®1

Sistemul de ecuații cu metoda potențialelor la noduri (V2 = 0):

Bazele Electrotehnicii I - Conf. dr. ing. Veronica PÂLTÂNEA, Facultatea de Inginerie Electrică 15
Sistemul de ecuații cu metoda potențialelor la noduri:

©i 3 ©i ©i
(l):Vi z^i— = ±=>Vi =
/?6 Rs R6 3

vi - o ©i _ ©
1 [Wb]
<Pz _ SR^^JL
lioag
30

Vx-0
<p3“ R6 38 [Wb]

„Rodiii
(2): = <p2 + <p3 = 2 38 (Wb]

Bazele Electrotehnicii I - Conf. dr. ing. Veronica PĂLTÂNEA, Facultatea de Inginerie Electrică 16
Intensitatea câmpului magnetic și inducția magnetică prin laturile circuitului;

Prin fiecare latură a circuitului magnetic valoarea inducției magnetice este:

<Pi = 2 = 2poWi
ag 38ag 38
__<p2 _ <p2 _ Pq/V^!
[T]
2 A ag 38
= <P3 = Mo^i
[T]
3 A ag 36

Bl Br 2p0Niii 2^1, , n
HS1 = [A/m]
MoMr Mo 38p0 38
b2 b2
b52 =
MoMr Mo
B3 b3
b83 = [A/m]
MoMr Mo

Bazele Electrotehnicii I - Conf. dr. ing. Veronica PĂLTÂNEA, Facultatea de Inginerie Electrică 17
Inductivitatea proprie a bobinei 1;

Inductivitatea proprie:

^i<Pi 2|i0a#N12i1 2poa<g^i2


fi 36

Inductivitatea de dispersie a bobinei 1:

/Vi<p2 = lipagN?
iA ~ 36

Ni<p3 = noagN}
q 38

Bazele Electrotehnicii I - Conf. dr. ing. Veronica PĂLTÂNEA, Facultatea de Inginerie Electrică 18
Energia magnetică dezvoltată în întrefier;

Energia magnetică dezvoltată în volumul întrefierului:

Pentru cele trei zone de întrefier, energia magnetică dezvoltată de circuitul magnetic este:
| HS1B, vr +1H82 B2 V2 +1H53 B3 V3

12N1i12[i0N1i1 o . e . l^iHPo^ili
2 36 36 -^6 + 2^6 36 ^6+2^6-^6“^6
N}i} 2\ioag l?Vi2if 1^1*1 ^«0 Nlll
6 9 2 6 9 2 6 9 6 3

L1i{ _ 2noagN}i}
2 36 2

Bazele Electrotehnicii I - Conf. dr. ing. Veronica PĂLTÂNEA, Facultatea de Inginerie Electrică 19
Forța magnetică dezvoltată în întrefier;

Utilizăm teorema forțelor generalizate pentru cazul curentului electric constant:

Pentru circuitul magnetic al problemei forța generalizată Xj este de tip forță newtoniană F, iar coordonata
generalizată Xj este lățimea întrefierului, care se poate modifica sub acțiunea forței.

Forță de atracție
3
f 9
362

Bazele Electrotehnicii I - Conf. dr. ing. Veronica PĂLTÂNEA, Facultatea de Inginerie Electrică 20

S-ar putea să vă placă și